Transcript
Page 1: implementacija konceptualnog testa iz nuklearne fizike i fizike čestica

SVEUČILIŠTE U ZAGREBU

PRIRODOSLOVNO-MATEMATIČKI FAKULTET

FIZIČKI ODSJEK

Dragan Borevac

Diplomski rad

IMPLEMENTACIJA KONCEPTUALNOG

TESTA IZ NUKLEARNE FIZIKE I FIZIKE

ČESTICA

Zagreb, 2011.

Page 2: implementacija konceptualnog testa iz nuklearne fizike i fizike čestica

SVEUČILIŠTE U ZAGREBU

PRIRODOSLOVNO-MATEMATIČKI FAKULTET

FIZIČKI ODSJEK

SMJER: PROF. FIZIKE

Dragan Borevac

Diplomski rad

IMPLEMENTACIJA KONCEPTUALNOG

TESTA IZ NUKLEARNE FIZIKE I FIZIKE

ČESTICA

Voditelj diplomskog rada: prof. dr. sc. Mirko Planinić

Ocjena diplomskog rada: ____________________

Povjerenstvo: 1.____________________

2.____________________

3.____________________

Datum polaganja: ____________________

Zagreb, 2011.

Page 3: implementacija konceptualnog testa iz nuklearne fizike i fizike čestica

Mojoj Biljani,

Jer si uvijek i bezrezervno vjerovala u mene

Mojoj Biljani,

jer si uvijek i bezrezervno vjerovala u mene!

Page 4: implementacija konceptualnog testa iz nuklearne fizike i fizike čestica

Hvala mentoru prof. dr. sc. Mirku Planiniću na pomoći i strpljenju, te

pruženoj potpori prilikom izrade ovog diplomskog rada.

Hvala dr. sc. Nikoli Poljaku na uloženom trudu, korisnim primjedbama,

sugestijama i savjetima tokom izrade diplomskog rada.

Hvala svima koji su sudjelovali u testiranju i tako doprinijeli istraživanju.

Hvala svim profesorima i asistentima s kojima sam se susreo tijekom svog

školovanja.

Hvala roditeljima koji su mi omogućili ovo školovanje, a posebno im

zahvaljujem na svoj ljubavi i podršci koju su mi pružili tokom studiranja.

Hvala!

Page 5: implementacija konceptualnog testa iz nuklearne fizike i fizike čestica

Sadržaj

Uvod ........................................................................................................................... 1

1. Povijesni razvoj fizike ........................................................................................... 2

1.1 Razlozi nastanka znanosti ................................................................................ 2

1.2 Razvoj fizike kroz povijest ................................................................................. 3

1.2.1 Rane civilizacije .......................................................................................... 3

1.2.2 Starogrčka fizika ......................................................................................... 3

1.2.3 Srednjovjekovna fizika ................................................................................ 4

1.2.4 Početak novog vijeka .................................................................................. 4

1.2.5 Temelji nove fizike ...................................................................................... 5

1.2.6 Izgradnja klasične fizike .............................................................................. 6

1.2.7 Suvremena fizika ........................................................................................ 7

1.3 Status prirodnih znanosti kroz povijest ............................................................. 8

2. Razvoj psihičkog života ...................................................................................... 10

2.1 Čimbenici razvoja ........................................................................................... 10

2.2 Kognitivni (spoznajni) ili intelektualni razvoj .................................................... 11

3. Nastava fizike ..................................................................................................... 12

3.1 Utjecaj Jeana Piageta ..................................................................................... 12

3.2 Učeničke intuitivne ideje ili pretkoncepcije - uočavanje njihovih uloga ............ 14

3.3 Konstruktivistički pristup nastavi fizike ............................................................ 15

4. Konceptualni test iz nuklearne fizike i fizike čestica ........................................... 16

4.1 Konstruiranje konceptualnog testa .................................................................. 16

4.2 Primjena konceptualnog testa ......................................................................... 16

4.3 Pregled dobivenih rezultata testiranja ............................................................. 17

4.4 Usporedba rezultata testiranja ........................................................................ 30

Zaključak .................................................................................................................. 32

Literatura .................................................................................................................. 33

Reference ................................................................................................................. 34

Prilog ....................................................................................................................... 35

Page 6: implementacija konceptualnog testa iz nuklearne fizike i fizike čestica

1

Uvod

Čovjek je u početku imao brojna pitanja na koja nije znao odgovore. Od samih

početaka, postojala je trajno prisutna znatiţelja i zanimanje za prirodu i prirodne pojave, a

znatiţelja je bila još veća ako su te pojave bitno utjecale na ţivot. Zahvaljujući toj znatiţelji

čovjek je krenuo u potragu za znanjem...

Sagledavši današnje znanje, kako fizike tako i cjelokupne znanosti, i površno

promotrivši njen razvitak, neupućenom pojedincu bi se na prvi pogled učinilo kao da su se

rješenja pojedinih problema odvijala jednosmjerno, u jednom odreĎenom pravcu, kao da

postoji put razvitka po kojemu su uvedeni fizikalni pojmovi. MeĎutim nije sve tako

jednostavno. Brojni temeljni fizikalni pojmovi su bili predmet ţučnih diskusija i razilaţenja

tijekom cijele povijesti, a mnogi od tih pojmova su i danas neriješeni do kraja.

Danas, kad fizika, kao i cjelokupna znanost, ne moţe dati odgovore na sva pitanja koja

bi čovjek htio znati, čovjekova znatiţelja je veća nego ikad. Zato je najveći i najambiciozniji

pokus u povijesti znanosti – Veliki hadronski sudarivač (LHC) – izazvao jako veliku paţnju,

ali i strah i iščekivanje. Razlog tolikoj uzbuĎenosti oko dosad najspektakularnijeg pokusa je

potraga za odgovorima na mnoga zanimljiva i vaţna pitanja, ne samo bitna za fizičare, nego

za dobrobit svih ljudi.

Usprkos brojnim odgovorima koje još nemamo, veliki opseg znanja nam je već

dostupan. Ta znanja se poučavaju u školi i na fakultetu. Znanja su se stoljećima samo

prenosila na nove generacije, a stečena znanja su omogućavala da čovjek cijeli ţivot uspješno

radi posao za koji se školovao. MeĎutim, danas je stanje potpuno drugačije: društvene

promjene su brţe, a brzina porasta znanja je jako velika. Zato više nije dovoljno da se

prirodne znanosti poučavaju na tradicionalan način, nego je potrebno traţiti nove načine

poučavanja. Danas se nastoji odgojiti i obrazovati čovjeka da iz školskih i studentskih klupa

izaĎe spreman za samostalan rad u modernom društvu. To je moguće samo ako stekne

operativno znanje, te razvije sposobnost za kritičko mišljenje.

Što se tiče implementacije suvremene fizike u sustavu obrazovanja, javljaju se

problemi usvajanja fizikalnih koncepata. Najveći problem čine čovjekove intuitivne ideje ili

predkoncepti, koji su duboko ukorijenjeni u čovjekovo razmišljanje. Kako bi se usvojilo

znanje i izgradili ispravni koncepti, bitno je prepoznati te predkoncepcije i smanjiti njihov

utjecaj.

Ovaj diplomski rad posvećen je istraţivanju najčešćih poteškoća kod učenja i

razumijevanja nuklearne fizike i fizike čestica na uvodnim kolegijima fizike na fakultetu.

Istraţivanje je provedeno ne samo kako bi se dobila opća slika o razumijevanju tih područja

fizike, nego kako bi se ubuduće razvili novi ili mijenjali već postojeći pristupi u poučavanju

tog fizikalnog sadrţaja.

Page 7: implementacija konceptualnog testa iz nuklearne fizike i fizike čestica

2

1. Povijesni razvoj fizike

1.1 Razlozi nastanka znanosti

Početak razvitka znanosti, a samim time i fizike, mora se traţiti u najstarijem razdoblju

ljudskog postojanja. Svako drukčije sagledavanje razvitka znanosti, koje bi početak uočavalo

na nekom razvijenijem stupnju razvitka čovječanstva, izazvalo bi dojam da su pojedine

znanstvene spoznaje nastale naglo. Dobro znamo da nije tako i da su fizikalna znanja nastala

postupno, u dugom procesu razvitka ljudskih aktivnosti, a kasnija su znanja samo logičan

nastavak već prijeĎenoga razvojnog puta. Početak znanosti nije odreĎen nekom godinom, pa

čak ni nekim kratkim vremenskim razdobljem. Početak znanosti smještamo u dugo razdoblje

stvaranja prvih pojmova, u vrijeme kad je čovjek pokušao riješiti bezbrojne probleme ţivota.

Čovjekova goloruka bitka s prirodom nije mu davala velikih nada da se u toj bitci

odrţi, ili se odrţavao uz velike gubitke. Bitna promjena nastaje onda kad je čovjek uvidio da

je ţivotinju lakše dotući nekom motkom pronaĎenom u prirodi nego se boriti s njom golih

ruku. TakoĎer je uvidio da motkom moţe lakše dohvatiti plodove na stablu nego penjanjem

po drvu ili čekanjem da plodovi padnu na tlo. Upotrijebljena motka je zapravo ništa drugo

nego produţena ruka, tako da čovjek ustvari nije smislio ništa posebno - samo je oponašao

prirodu. Ipak, ta rješenja nisu bila lagana i zahtijevala su ogroman napor, jer čovjek sigurno

nije odmah upotrijebio motku na najbolji mogući način. Moralo je biti bezbroj pokušaja dok

čovjek nije pronašao zadovoljavajuće rješenje. Čovjek je, stotinama, a i tisućama godina,

stjecao iskustva nalazeći sve bolja i prikladnija rješenja. Ta prva razdoblja su začeci nastanka

znanosti, koja je nastala prvenstveno iz praktičnih razloga - efikasnija borba s prirodom,

hranjenje, preţivljavanje, potrebni alati, oruĎa (motika, toljaga, kuka, poluga, luk i strijela...).

Čovjek je time riješio mnoge ţivotne probleme i tako si olakšao ţivot. Sve probleme je

pokušavao riješiti na različite načine, metodom pokušaja i pogreške, a najpovoljnije rješenje

je otkrio empirijski. Znao je npr. kako funkcionira poluga iako nije znao zašto je to tako.

Dakle, čovjek nije znao fizikalne zakone, ali ih je nesvjesno upotrebljavao jer je empirijski

spoznao njihovu praktičnu upotrebu.

Osim praktičnih razloga, znanost je nastala i zbog potrebe mjerenja. Kad je čovjek

počeo drţati domaće ţivotinje nije mu bilo svejedno hoće li neka od njih odlutati ili će ju

moţda pojesti divlja zvijer - tako je postupno naučio prebrojavati ţivotinje. TakoĎer, čovjek je

postupno prelazio sa nomadskog na sjedilački način ţivota, te se javila potreba za

premjeravanjem teritorija. Bitan razlog nastanka znanosti su i čovjekove primitivne

predodžbe. Ţiveći u prirodi, čovjek je opaţao mnoge prirodne pojave kojima nije znao točan

uzrok: izmjena dana i noći, izmjena godišnjih doba. Počeo je pratiti te cikličke pojave i

pokušavao ih opisivati.

Čovjek je kroz povijest stvarao različite predodţbu o svijetu. Npr. Sunce je kod svih

primitivnih naroda imalo vaţnu ulogu i obično su ga promatrali kao boţanstvo. Prirodne su

pojave oduvijek impresionirale ljude pa se neobičnim astronomskim pojavama često

pridavalo posebno značenje. Svemu onome što nije bilo redovito, npr. pomrčine, repatice,

oluje, munje, gromovi, pridavalo se veliko značenje.

U prvim erama čovjekova postojanja, znanja su bila ograničena na skup nepovezanih

činjenica. Ta su znanja bila konkretna i dobivena empirijski. Ipak, širina znanja je bilo puno

veća nego što se obično misli i to su zapravo prvi počeci nastanka i razvitka znanosti.

Page 8: implementacija konceptualnog testa iz nuklearne fizike i fizike čestica

3

1.2 Razvoj fizike kroz povijest

1.2.1 Rane civilizacije

Prijelaz iz prvih era čovjekova postojanja na prve civilizacije bio je postupan. Već u

tim prvim razdobljima prikupljeno je odreĎeno fizikalno, matematičko i astronomsko znanje.

Ono se u prvim civilizacijama dopunjuje i proširuje, ali se u početku ne razlikuje puno, što se

tiče konkretnog shvaćanja pojmova i empirijskog stjecanja znanja.

Rane civilizacije imale su zajedničku značajku da u znanosti ne formuliraju opća

pravila ni načela. Sve je rezultat empirijskih istraţivanja, odnosno opaţanja. Prve civilizacije

su gradile velike graĎevine pri čemu su upotrebljavali empirijska znanja o statici.

Upotrebljavali su se i strojevi čije je funkcioniranje bilo poznato iz iskustva i znalo se kako ih

treba upotrebljavati iako nije bilo poznato načelo na kojem se zasniva njihova iskoristivost.

Na temelju metode pokušaja i pogreške došlo se do saznanja da je teret lakše podići ako se

poveća krak koji se pritišće rukom, ali nitko se nije pitao zašto je to tako, pa se ne pojavljuju

ni odgovori. Postojale su i brojne primjene poluge poput vage i brodskog vesla.

Upotrebljavani su i drugi jednostavni strojevi poput kola i kolotura.

Što se tiče astronomskih znanja, ona su takoĎer bila empirijska. Astronomske pojave

su od davnina čovjeku djelovale impresivno, pa su ih promatrali i najprimitivniji narodi.

Zapazili su da se neke pojave ponavljaju, ali im nije bila poznata zakonitost zašto se to

dogaĎa. Nepoznavanje zakonitosti i uzroka pojedinih nebeskih pojava navela je stare narode

da im pripišu neka nadnaravna značenja. Svi su stari narodi imali svoju predodţbu svijeta i

predodţbu njegova ustrojstva. Sve su te predodţbe vrlo mistične i sve su proizašle iz

neposrednog promatranja svijeta koji ih okruţuje. Sve predodţbe su proizašle iz iskustva, ali

su bile interpretirane na mističan i religiozan način.

1.2.2 Starogrčka fizika

Iako se znanost, pa tako i fizika, a naročito mehanika i astronomija, javljaju vrlo rano,

njihov je pravi procvat u antičkoj Grčkoj, jer se tada počinju javljati nove ideje i pokušaji

razumijevanja prirode. Fizika je u to vrijeme najviše uznapredovala, no mnoga znanja i

koncepti o prirodi nisu tada samonikli nego su naslijeĎeni od starijih civilizacija, te često

interpretirana na nov način. Mnogi su začetnici fizikalne misli (Aristotel, Arhimed, Eratosten,

Tales, Leukip, Demokrit, Epikur...) roĎeni upravo u Grčkoj, u razdoblju 8. - 4. st. pr. Kr., pa

se smatra da su ondje udareni temelji učenju o prirodi kakve i danas poznajemo.

Najznačajniji je predstavnik toga razdoblja Aristotel koji je udario temelj razvoju

znanosti kakvu danas poznajemo. Aristotel razlikuje prirodna gibanja i nasilna gibanja za koja

je uvijek potrebna sila. Smatra da postoji razlika zemaljskog i nebeskog područja za koja

vrijede različite fizike. U tumačenju prirode Aristotel počinje koristiti pojmove koje danas

poznajemo kao fizičke veličine, kao put, brzina, otpor, a uvodi i mnoge koncepte značajne za

Aristotelovsku filozofiju, poput pokretačke sile, koncepta teţine i lakoće. Iako je Aristotelova

fizika bila kvalitativna i spekulativna, imala je velik utjecaj na razvoj znanosti.

Page 9: implementacija konceptualnog testa iz nuklearne fizike i fizike čestica

4

1.2.3 Srednjovjekovna fizika

Srednji vijek obiljeţavaju feudalni društveni odnosi i društveno ureĎenje te dominacija

kršćanskoga svjetonazora i religije u filozofiji. Razvoj fizike, i općenito prirodnih znanosti, u

srednjem vijeku zahvaljujemo arapskoj ili islamskoj civilizaciji koja od 8. do 15. st.

doţivljava svoje zlatno razdoblje.

Europa se u srednjem vijeku oslanja na Aristotelovo shvaćanje fizike, odnosno

prirodne filozofije. Crkva, kao dominantna institucija toga vremena, prihvaća i zastupa

Aristotelove teze, te su one općeprihvaćene (npr. kršćanstvu je dobro odgovarao geocentrični

poloţaj Zemlje). Krajem srednjega vijeka u Europi se javljaju nove ideje i pogledi na znanost

koji predstavljaju izravno sučeljavanje i kritiku Aristotelove filozofije prirode i stavova

Crkve.

U 6. st. indijski astronomi predlaţu heliocentrični sustav. U arapskoj civilizaciji u 9. st.

postavlja se hipoteza da se nebeska tijela i zvjezdane sfere vladaju po istim zakonitostima kao

i Zemlja. To je veliki napredak u odnosu na mislioce antičke Grčke koji su smatrali da se

„nebeska“ i „zemaljska“ gibanja odvijaju po različitim zakonitostima. Islamski znanstvenici

razlikuju masu i teţinu, tvrde da zrak ima teţinu i da mu je gustoća manja od gustoće vode.

Kritiziraju Aristotela i njegovu ideju o stalnoj sili koja je potrebna da bi odrţala gibanje tijela.

Dok arapski svijet proţivljava svoje zlatno razdoblje, Europa je pod utjecajem

skolastike i Crkve. Biblija i Aristotelova tumačenja su uzimana kao nepogrešiva, no ipak se

javljaju novosti i napredci, naročito u znanstvenom razmišljanju. U 13. st. protiv nekritičkih

prihvaćanja Aristotelovih tumačenja ustaje engleski franjevac Roger Bacon koji smatra da

pokus treba biti temelj znanosti i jedina mjerodavna metoda u donošenju zaključaka. Zbog

svojih je naprednih stavova Bacon izbačen sa sveučilišta, osuĎen kao heretik i utamničen. U

srednjovjekovnoj Europi je astronomija uţivala osobitu popularnost jer se smatrala

interpretiranjem Boţjih zakonitosti. Njome su se bavili crkveni učenjaci i duţnosnici, jedini

školovani ljudi toga vremena. Vjerovalo se u geocentrični model svemira utemeljen na

Ptolomejevim učenjima, koji je zastupala Crkva te se astronomija u srednjevjekovnoj Europi

dugo svodila na doradu Ptolomejeva modela i nije donosila nova otkrića.

1.2.4 Početak novog vijeka

Od 14. do 16. stoljeća nastupa vrijeme jednoga od najznačajnijih pokreta u kulturi

zapadne Europe, koji je označio završetak srednjega vijeka i doveo do preokreta u znanosti,

filozofiji i umjetnosti. To je vrijeme renesanse ili preporoda. U renesansi se razvijaju gradovi,

trgovina i promet, raste broj stanovnika, povećava se značaj novca, nastaju prve manufakture

te raspadom feudalnoga nastaje novo, graĎansko društvo. U renesansi se raĎa racionalno i

znanstveno shvaćanje svijeta. Nosioci su nove renesansne kulture bili humanisti koji su

kulturu usmjerenu prema čovjeku suprotstavljali skolastičkoj znanosti i teologiji. Središte

univerzuma više nije Bog, nego čovjek – univerzalno obrazovani humanist, predstavnik

novčane i intelektualne elite. Renesansa je vrijeme velikih otkrića; istraţivači Kristofor

Kolombo, Vasco da Gamma, Ferdinand Magellan i dr. su otkrili nove kontinente i nove

zemlje. Brojnim je istraţivačkim pothvatima moreplovaca dokazano da je Zemlja okrugla

čime je omogućeno novo shvaćanje svemira.

Page 10: implementacija konceptualnog testa iz nuklearne fizike i fizike čestica

5

U matematici i fizici renesansa ne donosi nove velike spoznaje, ali je ovo razdoblje

značajno jer nagoviješta prekretnicu u razvoju znanosti i kulture. Poslije tisućljetnoga zastoja,

Aristotelova prirodna filozofija doţivljava prve odlučne kritike, pa i potpuno negiranje

vrijednosti. Znanstvenici započinju borbu protiv autoriteta, protiv dominacije Aristotela i

skolastičkoga pristupa znanosti. Nastaju nove ideje istaknutih pojedinaca kao što su Kopernik,

Kepler, Galileo i drugi. Renesansni učenjaci obnavljaju i počinju ponovno cijeniti učenja

antike i civilizacija Istoka. Započete su pripreme za novo doba.

Veliki doprinosi renesanse nastaju u domeni astronomije i konačno dovode do

napuštanja tisućljetnih učenja Aristotela i Ptolomeja te razvijanja novoga, temeljno različitoga

pogleda na mjesto čovjeka u svemiru. Nikola Kopernik zagovara heliocentrični model

svemira. UtvrĎuje postojanje trostrukoga gibanja Zemlje: njezinu rotaciju oko osi, revoluciju

(tj. gibanja Zemlje oko Sunca) i precesiju Zemljine osi. Kopernikovim je učenjem bio

ponesen i mladi Giordano Bruno. Izlagao je svoje učenje o svemiru i planetnim sustavima

drugih zvijezda te istaknuo ideju o jedinstvenim zakonima prirode. Nešto slično postoji i

danas, kad se nastoji ujediniti četiri fundamentalne sile.

1.2.5 Temelji nove fizike

Nakon renesanse u 17. st. nastupa u zapadnoeuropskoj kulturi tzv. vrijeme

racionalizma. Ono prethodi pokretu prosvjetiteljstva koje je obiljeţilo 18. st. U 17. st. su

postavljeni temelji klasične fizike. „Ocem fizike“ i „svjetlom novog doba“ često se naziva

Galileo Galilei, čovjek koji je počeo izgraĎivati novu fiziku temeljenu na pokusu i

matematičkom opisu. To je čovjek koji je prvi uveo eksperimentalnu metodu, pokus

ustanovio metodom znanstvenoga istraţivanja i započeo matematičko formuliranje fizikalnih

zakona, pa se zato smatra osnivačem klasične fizike i mehanike. U to vrijeme djeluje i, po

mnogima, najveći fizičar i znanstvenik koji je ikad ţivio Isaac Newton. Kulminacija je fizike

17. st. bila izdavanje njegove Principie iz 1687. godine u kojoj su definirani temeljni pojmovi

i formulirana nova slika svijeta. Jedan od najvećih znanstvenih genija svih vremena Isaac

Newton, svojim je djelom Matematička načela prirodne filozofije zauvijek promijenio

mehaniku i cjelokupnu fiziku. Svoje izučavanje mehanike Newton utemeljuje na poznavanju

djela slavnih prethodnika; objedinjuje spoznaje Galileija i Keplera u jednu teoriju gravitacije i

postavlja temelj klasičnoj mehanici. Definira ključne pojmove mehanike: masu, količinu

gibanja, silu i tromost, te formulira tri osnovna zakona gibanja. Koristi latinski pojam gravitas

za učinak koji danas zovemo silom teţom i definira zakon opće gravitacije. Iako je mnogima

zvučalo suviše tajanstveno, Newton postulira gravitaciju kao nevidljivu silu koja djeluje na

daljinu. Pojam sile kakav danas poznajemo, kao djelovanje koje mijenja stanje gibanja tijela,

proizlazi iz Newtonova koncepta sile, iz Newtonove dinamike.

Najvišim načelom svakoga znanstvenog istraţivanja se smatra pokus, a metoda koja se

predlaţe za dolazak do novih spoznaja jest metoda indukcije koja zapravo predstavlja

jedinstvo eksperimenta i racionalne spoznaje, no prije njezine primjene treba očistiti razum od

nagomilanih predrasuda. Teţi se „velikoj obnovi“ (temeljitoj preobrazbi znanosti i ţivota

uopće) i drţi se da je cilj znanosti i filozofije opskrbiti ljudski ţivot novim pronalascima i

dobrima, ovladati prirodom i povećati čovjekovu moć nad prirodom. Kritizira se dotadašnje

filozofske i znanstvene misli i upozorava na potrebu revizije pojmova i metoda kojima su se

gradile znanstvene teorije. Smatra se da osnova spoznaje treba biti mogućnost čovjeka da

svojim umom unosi red u proučavanje stvari te onda pravilno zaključuje. Ističe se vaţnost

logičke analize, preciznoga zaključka utemeljenoga na jasnim i jednostavnim načelima.

Page 11: implementacija konceptualnog testa iz nuklearne fizike i fizike čestica

6

Prema legendi, na razmišljanje o gravitaciji Newtona je navela jabuka koja mu je pala

na glavu. U svojim Načelima iznosi da Zemlja privlačnom silom djeluje na Mjesec i otklanja

ga od pravocrtne putanje. Ta sila, smatra Newton, opada s kvadratom njihovih udaljenosti. Isti

zaključak primjenjuje za gibanje planeta uzrokovano privlačenjem Sunca. Prema trećem

zakonu gibanja, i planeti privlače Sunce. Usporedbom ubrzanja sile teţe i Mjesečeva

centripetalnoga ubrzanja zaključuje da slobodni pad i kruţenje Mjeseca oko Zemlje imaju isti

uzrok. Newton demonstrira konzistentnost Keplerovih zakona gibanja planeta i svoje teorije

gravitacije te tako pokazuje da se nebeska i zemaljska gibanja odvijaju po istim

zakonitostima. To je bilo prvo „ujedinjenje sila“ u povijesti fizike koje je otklonilo posljednje

ostatke aristotelovske fizike i sve dvojbe oko heliocentrizma.

1.2.6 Izgradnja klasične fizike

Analitičke se metode mehanike, razvijene u 18. st., tijekom 19. st. počinju

primjenjivati i na istraţivanje fizikalnih pojava. Matematizirani koncepti o energiji u drugoj

su polovici 19. st. uzrokovali do tada najveću ekspanziju fizike, kao i preispitivanje

tradicionalnih ideja o fizičkom svijetu. Newtonova fizika dobiva naziv klasična fizika. U

fizici 19. stoljeća teţilo se objašnjenju svih fizikalnih područja pomoću mehanike, a sve se

više stvaralo uvjerenje da je 19. stoljeće steklo konačna i sigurna znanja u fizici koja se više

neće mijenjati nego samo dopunjavati novim istraţivanjima. Smatralo se da su pronaĎeni

fizikalni zakoni, zajedno s onima koji će se tek pronaći, dovoljni za odreĎenje budućeg

razvitka svijeta.

19. stoljeće je vrijeme teorija i eksperimentalnih doprinosa o izmjenjivosti mehaničke,

kemijske, električne energije i topline, odnosno rada. Oblikuju se zakoni termodinamike,

uvodi pojam entropije i utemeljuje statistička fizika. TakoĎer, na samome početku 19. st.

dolazi do spoznaje o povezanosti električnih i magnetskih pojava, što je nakon Newtonova

ukidanja Aristotelove „nebeske i zemaljske fizike“ vodilo prema drugom velikom ujedinjenju

sila u povijesti fizike. Uvodi se koncept polja i uobličuje klasična elektrodinamika. Vrhunac

fizičkih teorija, a ujedno i uvod u elektrotehničku revoluciju, bio je koncept svjetlosti kao

prijenosa energije u vidu elektromagnetnog vala, te otkriće elektromagnetnog zračenja.

Najveći mislioc tog vremena je zasigurno James Clerk Maxwell, koji po svojim zaslugama i

otkrićima stoji uz bok Newtonu. Otkriće zakona elektromagnetizma i elektromagnetskih

valova, kao i njegovo statističko tumačenje ustrojstva tvari, temelj su gotovo cjelokupnog

današnjeg tehnološkog napretka: od novih vrsta materijala do suvremene računalne i

komunikacijske visoke tehnologije.

Fizičari su se sredinom 19. stoljeća različito odnosili prema problemu sile. Bilo je onih

koji su poricali postojanje sile, ali i onih koji su zastupali njezino postojanje, i to kao

fundamentalnog pojma u fizici. Fizičari su se zapravo dijelili na dinamiste i mehaniciste.

MeĎutim, čak su i mehanicisti izvodili Newtonov izraz za opću gravitaciju iz gibanja čestica,

pa su tako formalno upotrebljavali isti izraz kao i dinamisti, ali su istodobno samo drukčije

tumačili fizikalne veličine koje su ulazile u račun. Na taj način su zapravo i jedni i drugi, iako

su se bitno razlikovali u svojim temeljnim koncepcijama, formalno gledajući izgraĎivali istu

klasičnu fiziku.

Page 12: implementacija konceptualnog testa iz nuklearne fizike i fizike čestica

7

Osim izgradnje klasične fizike imali su još jednu zajedničku značajku: i jedni i drugi

su vjerovali da se sve fizikalne pojave mogu objasniti pomoću mehanike. I jedni i drugi su

smatrali da nova fizikalna otkrića potvrĎuju misao o mehaničkom tumačenju svih fizikalnih

pojava.Takvi pogledi na fiziku, prirodne znanosti, kao i na znanost uopće, uvjetovali su

sasvim nov odnos prema sigurnosti znanja i prema razvitku znanstvenih ideja. Tadašnji

znanstvenici smatraju kako je njihovo znanje, iako nepotpuno, sigurna istina. To znanje treba

samo dopunjavati i time će se dolaziti sve bliţe potpunom apsolutnom znanju.

1.2.7 Suvremena fizika

Krajem 19. stoljeća fizika je trijumfirala u svojoj uvjerenosti da se postignuta znanja

neće osporiti. Fizičari su bili neograničeno samouvjereni u sigurnost svojih znanja. Smatralo

se kako su svi vaţni problemi fizike riješeni te da fizičari nemaju osobito što više za otkriti.

Ali takva radost bila je kratkotrajna, već početkom 20. stoljeća uslijedila su neka otkrića koja

su pokolebala tadašnju sigurnost i samouvjerenost, te donijela povijesne prevrate u shvaćanju

prirode. Najveći od njih su bili teorija relativnosti i kvantna fizika. Kasnije je uslijedio još

veći, eksponencijalni razvoj spoznaja iz fizike i znanosti općenito, te je 20. stoljeće razdoblje

mnogih tehnoloških revolucija.

Albert Einstein je 1905. u samo nekoliko mjeseci objavio pet kapitalnih znanstvenih

radova. U specijalnoj teoriji relativnosti proširio je zakone gibanja na pojave pri velikim

brzinama, bliskima brzini svjetosti. Istodobno je otkrio i postojanje formule za energiju

mirovanja tijela, ovisnu o masi, E0 = mc2, koja skriva tajnu goleme nuklearne energije. Osim

toga, poopćio je Planckovu ideju o kvantizaciji energije: postavio je hipotezu da je svako

elektromagnetsko zračenje kvantizirano, tj. da ima osnovne kvante energije – fotone. Tom je

smjelom pretpostavkom potvrdio kvantnu hipotezu i postavio temelje kvantnoj fizici.

Objasnio je i Brownovo gibanje i time udahnuo vjerodostojnost svim ranijim hipotezama o

atomskom i molekulskom ustrojstvu tvari. Svoj doprinos fizici okrunio je općom teorijom

relativnosti, u kojoj je zakone gravitacijskog privlačenja povezao sa svojstvom inercije, i time

otvorio put suvremenoj astrofizici i kozmologiji, novom poglavlju u fizici, posvećenom

svojstvima svemira kakvog danas poznajemo, ali i njegovoj prošlosti i budućnosti.

Tijekom ovoga stoljeća došlo je i do velikih otkrića o strukturi atoma: 1911. godine

Ernest Rutherford otkriva atomsku jezgru sastavljenu od pozitivnih protona, a 1932. godine

James Chadwick otkriva neutron.

Početkom su 20. st. Max Planck, Albert Einstein, Niels Bohr i drugi razvili temeljne

elemente kvantne teorije da bi objasnili nekonzistentnosti nekih fizikalnih eksperimenata, a

1925. godine su Werner Heisenberg i Erwin Schrödinger formulirali kvantnu mehaniku.

Razvoj je kvantne mehanike tijekom 20. st. doveo do stvaranja moćnih teorijskih alata za

nastanak i razvoj novih područja fizike. Fizika čvrstoga stanja počinje izučavati fizička

svojstva kristala i tekućina, kristalne strukture te, nešto kasnije, poluvodiče i pojavu

supravodljivosti.

Prije Drugoga svjetskog rata i za vrijeme njegova trajanja provode se intenzivna

istraţivanja u području nuklearne fizike s ciljem dobivanja nuklearnoga oruţja. Saveznički

projekt Manhattan, predvoĎen Enricom Fermijem, prvi je ostvario taj cilj: 1942. godine

postignuta je prva nuklearna lančana reakcija, a 1945. godine je u New Mexicu (SAD)

izvedena prva atomska eksplozija.

Page 13: implementacija konceptualnog testa iz nuklearne fizike i fizike čestica

8

Tijekom 20. st. se razvija i kvantna teorija polja, koja ujedinjuje kvantnu mehaniku i

specijalnu teoriju relativnosti. Svoj suvremeni oblik dostiţe polovicom 20. st. u radovima

Feynmana i drugih. Kvantna je teorija polja osigurala kvalitetan okvir za razvoj fizike

elementarnih čestica, koja izučava fundamentalne sile i elementarne čestice. Godine 1954.

Yang i Mills postavljaju temelje koji dovode do tzv. standardnoga modela, upotpunjenoga u

drugoj polovici 20. st., koji uspješno opisuje tri od četiri fundamentalne interakcije izmeĎu

elementarnih čestica od kojih se sastoji sva poznata materija (model ne uključuje gravitaciju).

1.3 Status prirodnih znanosti kroz povijest

Znanost je prolazila kroz različite faze svojeg razvitka, naučne teorije su se mijenjale,

pogledi na probleme i njihovo rješavanje su se usavršavali i mijenjali svoj opseg. Baš kao i

znanost općenito, nastava fizike je takoĎer prolazila svoje faze.

Starim Grcima i Rimljanima nije bio cilj da se učenici bave dubljim proučavanjem

fizikalnih pojava već je njihov odgojni ideal bio stvoriti drţavnika. MeĎutim, bitno je da je

već tada sam odgoj shvaćen kao najvaţniji društveni zadatak. Kod Grka je fizika pomagala pri

stvaranju filozofskih tema, a naučeno se prenosilo usmenom predajom. Tek su ih rijetki

zapisivali u svojim djelima kao na primjer Aristotel. Kasnije se vaţnost fizike kao znanosti

promijenila pa su stari Rimljani gradili filozofske, medicinske i graditeljske škole u kojima se

poučavala fizika. Štoviše, upošljavali su posebne učitelje te se pobrinuli i za posebne

prostorije u kojima se takva nastava odrţavala. Općenito, Rimljani su se zalagali za tzv.

„radosnu nastavu“ i smatrali su da djecu treba dobro upoznati, individualno im pristupati,

voljeti ih, pohvaljivati. Javljaju se prve ideje kako gradivo treba dijeliti na smislene cjeline.

U srednjem vijeku otišlo se nekoliko koraka u nazad. Propast Rimskog Carstva i

prevlast Crkve nije povoljno utjecala na razvitak prirodnih znanosti. Učenik je morao preuzeti

ulogu dobrog i poslušnog kršćanina koji ne postavlja previše pitanja nego prihvaća crkvene

dogme. Učitelji/odgajatelji su bili svirepi, a samo učenje se svodilo na mehaničko

zapamćivanje.

No već se u 16. stoljeću kidaju okovi crkvenog dogmatizma, a Aristotel više nije jedini

autoritet. Pomalo se prihvaćaju i Kopernikova poučavanja. Javljaju se sve odlučniji zahtjevi

za uvoĎenjem prirodnih znanosti u škole, no trebalo je proći još dosta vremena dok se to i ne

obistini pod pritiskom bogatog sloja društva kojem je bio potreban obrazovani kadar.

Prekretnicu je nagovijestilo razdoblje renesanse. MeĎutim, iako tada nastaju vaţne

nove ideje istaknutih pojedinaca, prirodne su znanosti i dalje teško nalazile svoje mjesto u

društvu. Zbog neslaganja s tada postojećim dogmama brojni istaknuti pojedinci našli su se na

udaru vladajućih društvenih struktura, poglavito Crkve, a njihova su djela bila zabranjivana.

Sredinom 17. stoljeća, nakon dugog razdoblja nemira, ratova, poţara i epidemija, stanje u

Europi se donekle sredilo. Došlo se do konsenzusa da učitelje treba znanstveno educirati.

Osniva se prva znanstvena akademija 1659. u Toskani, 1662. osniva se Kraljevsko društvo u

Londonu, 1666. Akademija znanosti u Parizu i taj val se proširio cijelom Europom. Cilj

nastave je bio razumjeti gradivo, a vodilo se računa o osobinama djece i njihovoj aktivnosti u

nastavi. Već su tada neki humanisti uvidjeli kako je krajnja svrha nastave zapravo razvijanje

dječjeg mišljenja, a glavno načelo bi nastave bi trebalo biti osposobljavanje za

samoobrazovanje i utilitarizam. Ipak, te ideje su se počele realizirati tek krajem 18. stoljeća.

Page 14: implementacija konceptualnog testa iz nuklearne fizike i fizike čestica

9

Krajem 18. st. razvila se tehnologija koja je nastojala steći moć nad ljudskom

produktivnošću i njome ovladati. To je dovelo do industrijske revolucije i jačanja tehnoloških

institucija u društvu. Sve uspješnija tehnologija počinje pretendirati na onu društvenu poziciju

koju je filozofija prirode stjecala prethodnih 200 godina. Ta nova situacija, nastala

industrijskom revolucijom, i pokušaj podreĎivanja filozofije prirode tehnologiji, postavila je

pred filozofiju prirode zadaću da redefinira svoje područje rada i ponovno uspostavi svoju

ulogu u društvu. Uvedeni su novi nazivi prirodne znanosti i prirodoznanstvenik umjesto

filozofija prirode i filozof prirode, utvrĎena su stroga pravila i standardi za one koji ţele

postati prirodoznanstvenici. Prirodoznanstvenici su se povlačili u osamu sveučilišta. Proces

redefiniranja je dovršen do 1860. kad je fizika, zajedno s kemijom, biologijom i geologijom,

učvršćena kao disciplina i kao administrativna jedinica unutar sveučilišta. Na taj način je

filozofija prirode preoblikovana u profesionalizirane prirodne znanosti koje su usredotočile

nastojanja na intelektualnu znatiželju i na znanje zbog samoga znanja. Istodobno su

prirodoslovni predmeti uvedeni u javne škole, u više razrede gimnazija. Njihovi kurikulumi su

modelirani u skladu sa svjetonazorom prirodnih znanosti: znanje poradi znanja, ignoriranje

praktičnih znanja, zanemarivanje uloge prirodnih znanosti u društvu i njihove odgovornosti

spram društva. Svrha poučavanja prirodnih znanosti bila je pripremanje učenika za studij tih

znanosti na sveučilištu. U kurikulumima se naglašava čista apstrakcija i demonstrira estetsko

jedinstvo discipline, dok su praktično znanje i društveni aspekti sasvim isključeni. Problem je

u tome što je u nekim sredinama tako i danas.

Tek u 19. stoljeću počinje nastava fizike bazirana na promatranju i eksperimentiranju.

No provoĎenje te nastave i dan danas zapinje. Početkom prošlog stoljeća problem je bio

nedostatak kvalificiranog kadra jer fakulteti nisu pripremali studente za takvu vrstu nastave.

To je vrijeme kad se uvidjelo da je za učenje bitan razvoj misaonih sposobnosti i motivacije,

te da treba kombinirati aktivno učenje (temeljem vlastita iskustva) i pasivno učenje (u školi).

TakoĎer se uvidjela nuţnost individualnog pristupa svakom djetetu, kao i sloboda nastavnih

oblika i metoda.

U 20. stoljeću nastaje nagli i uspješan razvoj prirodnih znanosti, ali i ubrzane

društvene promjene. Prirodne znanosti i tehnologija povezale su se u novu društvenu snagu

pod nazivom istraživanje i razvoj. Prirodne znanosti dobivaju velika sredstva, partnerstva s

vladom, vojskom i industrijom. Smanjuje se procijep izmeĎu „čiste“ i „primijenjene“

znanosti. Razvoj visoke tehnologije zasniva se na rezultatima prirodnih znanosti.

Danas znanstvenici postaju spremniji primijeniti svoje znanje u rješavanju svjetskih

problema. Sve veće društveno značenje prirodoznanstvenog znanja današnja je realnost.

Prirodne se znanosti razvijaju u svijetu meĎudjelovanja politike, ekonomije i rata.Čistom

znanošču bavi se tek mali dio akademskih znanstvenika, a i oni ne mogu izbjeći političko

lobiranje za dobivanje projekata i sredstava. Današnji školski prirodoznanstveni predmeti

odišu akademskim idealizmom 19. stoljeća, što nije u skladu s realitetom u kojemu

funkcioniraju prirodne znanosti poslije Drugoga svjetskog rata. To stanje se nastoji

promijeniti. Svrha poučavanja prirodnih znanosti postaje prirodoznanstveno opismenjivanje

ukupne populacije i jačanje njihove sposobnosti za djelovanje. U tom se smislu radi na

kurikularnim reformama u kojima preteţe ideja tzv. Science-technology-society pristupa,

putem kojega se u nastavi prirodnih znanosti pokušava naglasiti povezanost i meĎudjelovanje

prirodnih znanosti, tehnologije i društva.

Page 15: implementacija konceptualnog testa iz nuklearne fizike i fizike čestica

10

2. Razvoj psihičkog života

Sposobnosti i značajke čovjekova ponašanja rezultat su dvaju razvojnih procesa:

razvoja ljudske vrste i razvoja pojedinca od njegova začeća.

Razvoj ljudske vrste tekao je milenijima u obliku postupnog usavršavanja i

specijalizacije dijelova ljudskog organizma. Procesima adaptacije, tj. prilagoĎavanja na

promjenjive vanjske okolnosti i odabiranja za opstanak samo najprilagoĎenijih oblika

usavršavalo se ponašanje do onih oblika koje danas poznajemo kao najviši stupanj prilagodbe

u ţivom svijetu – ljudsko ponašanje.

Ponašanjem ţivotinja preteţito upravljaju instinkti. To su gotove, sloţene radnje u

kojima su unaprijed, biološki utvrĎeni oblici ponašanja i situacije koje će ih izazvati.

Sezonska seoba ptica, gradnja gnijezda, „podjela rada“ u pčelinjaku, briga za mladunčad, sve

su to sloţene, ali priroĎene, nenaučene radnje, reakcije organizma ţivotinje na okolne

promjene (temperatura) ili promjene u organizmu (razina hormona i sl.). Ljudskim

ponašanjem ne upravljaju takvi gotovi modeli ponašanja koji bi bili već „ugraĎeni“ u ţivčani

sustav u trenutku roĎenja. Najveći je dio ljudskog ponašanja elastičan, promjenjiv,

prilagodljiv situaciji i zato se u ljudski ţivčani sustav tek moraju ugraditi sloţeni modeli

ponašanja koji su rezultat iskustva i ponavljanja iskustva – učenja.

Razvoj čovjeka (kao ljudske jedinke) od njegova začeća bitno je drugačiji od razvoja

ţivotinjskog mladunčeta, i to ne samo zbog prednosti u savršenijoj biološkoj strukturi, već i

zbog specifičnosti ljudskog iskustva.

Iskustvo koje stječe pojedinac tijekom svoga individualnog razvoja jest socijalno

iskustvo. Za razliku od ţivotinjkog svijeta u kojemu svaka pojedinačna ţivotinja mora iznova

razvijati svoj biološki zadani repertoar i svaka generacija počinje „od početka“, ljudsko se

iskustvo prenosi i ono se zato kumulira: svaka generacija zna više od prethodne i prenosi to

iskustvo na svakog pojedinca tijekom njegova individualnog razvoja. Taj prijenos se ostvaruje

poučavanjem.

2.1 Čimbenici razvoja

Pitanje o naslijeĎenosti ili stečenosti ljudskih osobina vaţno je ne samo za psihologiju

ili pedagogiju, nego i za svakodnevni praktični odgoj i obrazovanje. Ako su osobine pojedinca

potpuno ili najvećim dijelom odreĎene nasljedno, znači da dijete u trenutku roĎenja nosi u

sebi gotov nacrt budućih osobina. Nikakva nastojanja okoline neće moći promijeniti takav

gotov nacrt. S druge pak strane, ako se dijete raĎa kao „čista ploča“ na kojoj će tek njegovo

buduće iskustvo ostaviti tragove i tako ga oblikovati, samo je okolina odgovorna za ono što će

se iz djeteta razviti. Iz takvih razmišljanja o odgovornosti ili slobodi od odgovornosti društva

za razvoj čovjeka, dolaze dva meĎusobno oprečna stava. Nativizam (lat. nativus = uroĎen) je

shvaćanje da je cjelokupan psihofizički razvoj posljedica naslijeĎenosti. Prema njemu su sve

buduće djetetove osobine već odreĎene u trenutku njegova začeća. Hoće li se u djeteta razviti

odreĎene osobine ovisi samo o tome jesu li djetetovi preci i roditelji posjedovali te osobine, a

ništa ili malo moţe se postići odgojem i obrazovanjem.

Page 16: implementacija konceptualnog testa iz nuklearne fizike i fizike čestica

11

Suprotno stanovište je empirizam (grč. empeiria = iskustvo). To je shvaćanje da su sve

osobine koje će dijete steći posljedica njegova iskustva. Prema tome, ono što će odrediti

djetetove osobine je: priroda dječjeg iskustva, kvaliteta okoline u kojoj dijete raste,

mogućnosti za učenje i vrste stimulacije koje okolina pruţa.

U suvremenoj znanosti prevladava stanovište, utemeljeno na istraţivanju, da u razvoju

čovjeka vaţnu ulogu ima istodobni utjecaj i nasljednih i okolinskih čimbenika, te da je njihov

odnos čak interakcijski. Svaki je pojedinac upravo zato toliko jedinstven i neponovljiv jer se

razvijao pod utjecajem različitih kombinacija specifičnih naslijeĎenih karakteristika (graĎa

organizma, tipična podraţljivost i osjetljivost ţivčanog sustava, elastičnost i prilagodljivost

ţivčanog tkiva) i uvjeta za razvoj (neposredna društvena okolina: roditelji, braća i sestre,

vršnjaci, susjedstvo, učitelji, nastavnici i profesori; šira društvena zajednica: društvo u

najširem smislu, društveno-ekonomski odnosi, tip društvenog ureĎenja). Ono što nasljeĎe

pridonosi razvoju jesu mogućnosti razvoja. To znači da su nasljeĎem odreĎene granice do

kojih se pojedinac moţe razviti, a utjecaji okoline odredit će u kojoj će se mjeri potencijalni

nacrt razvoja ostvariti. Prema tome, pogrešno je pitati „Koji je čimbenik bitan – nasljeĎe ili

okolina?“. Da napravimo kruh, treba nam tijesto i visoka temperatura. Besmisleno je pitati je

li za kruh vaţnije tijesto ili pečenje. Tako su i jedna i druga skupina čimbenika nuţni uvjeti

razvoja.

Osim utjecaja naslijeĎa i okoline, za razvoj sposobnosti i osobina pojedinca, vaţna je i

čovjekova aktivnost. U prvim godinama čovjekova ţivota, tipična aktivnost je baratanje

predmetima, u kasnijoj dobi tipična aktivnost je igra, a u školskoj stjecanje pojmova i

organizirano učenje.

2.2 Kognitivni (spoznajni) ili intelektualni razvoj

Sposobnost spoznavanja vanjskog svijeta, uočavanje veza i odnosa meĎu stvarima i

pojavama, te mogućnost rješavanja problema i prilagoĎavanja na nove i promjenjive situacije,

postupno se stječe i usavršava od roĎenja do kraja adolescencije. Taj razvoj obuhvaća

nastajanje i usavršavanje mnogih psihičkih funkcija i naziva se jednim imenom kognitivni

(spoznajni) ili intelektualni razvoj.

Brojna proučavanja kognitivnog razvoja dala su rezultate koji uglavnom govore o

tome kada djeca ovladavaju misaonim funkcijama (kao što su npr. stjecanje pojmova,

definiranje pojmova pomoću višega rodnog pojma, tj. pojma klase, mogućnost apstrakcije i

generalizacije, hipotetičnost i apstraktnost mišljenja itd.). Ovi rezultati pokazuju da postoje

velike sličnosti u razvoju dječjeg mišljenja u djece različitih naroda i socijalnih sredina.

Najpotpuniji prikaz razvoja dječjeg mišljenja dao je Jean Piaget koji opisuje četiri

jasno odreĎene faze i velik broj podfaza u njihovu sklopu. U svakoj je fazi sadrţan opis onoga

što djeca odreĎene dobi mogu ili ne mogu dokučiti mišljenjem, tj. kojim misaonim

operacijama mogu baratati. O osnovnim značajkama mišljenja u četiri glavne Piagetove faze

više u idućem poglavlju.

Page 17: implementacija konceptualnog testa iz nuklearne fizike i fizike čestica

12

3. Nastava fizike

U drugoj polovini 19. stoljeća fizika se uvodi u školu kao zaseban predmet. Paralelno s

nastavom fizike razvija se i disciplina pod nazivom metodika nastave fizike. Njena temeljna

problematika je istraţivanje i pronalaţenje učinkovitih načina prirodoznanstvenog

opismenjivanja ukupne populacije, a posebno učenika, te istraţivanje meĎudjelovanja

prirodnih znanosti, tehnologije i društva.

Tradicionalno se fizika poučavala kao mnoštvo manje ili više meĎusobno povezanih

činjenica i informacija. MeĎutim, pedesetih i šezdesetih godina 20. stoljeća postalo je jasno

da, čak ako i pretpostavimo da dio učenika stekne izvjesno deklarativno znanje, to nije

dovoljno. Ubrzo je postalo jasno da je takav tradicionalni način poučavanja daleko od

optimalnog, pa dolazi do velikog zaokreta u metodici nastave fizike. Javlja se ideja o nuţnosti

interakcije u učenju i poučavanju, a prvi korak u tom smjeru je načinio švicarski psiholog

Jean Piaget u svojoj teoriji kognitivnog razvoja. Slijedeći je korak bila spoznaja da veliku

ulogu u učenju imaju učeničke intuitivne ideje ili pretkoncepcije. Treći korak je pojava

edukacijskog konstruktivizma, koji povezuje i usustavljuje Piagetove ideje i problematiku

pretkoncepcija te ih povezuje s postignućima filozofije prirodnih znanosti.

3.1 Utjecaj Jeana Piageta

Početak velikih promjena u pristupu problematici nastave fizike sredinom 20. stoljeća

izravno je povezan s utjecajem ideja Piagetove teorije kognitivnog razvoja djeteta. Piaget je

za svoja istraţivanja o razvoju mišljenja djece rabio pitanja i testove s fizikalnom

problematikom, i zato su njegove ideje jako utjecale na metodičare fizike. Piaget je i

utemeljitelj modernog edukacijskog konstruktivizma i time je njegov utjecaj na suvremenu

metodiku fizike još i veći.

Jedan je od najvaţnijih Piagetovih zaključaka da je za kognitivni rast pojedinca bitno

njegovo aktivno mentalno sudjelovanje u razrješavanju predočenih problemskih situacija.

Kognitivna struktura moţe se razvijati samo na temelju već postojeće, a to se postiţe daljnjom

dogradnjom znanja ili restrukturiranjem postojeće mentalne strukture. U tom razvoju jednu od

temeljnih uloga ima društvena transmisija (kasniji termin: interakcija). Razlog tomu je što se

pokazalo da dijete operira iz izrazito egocentričnog referentnog sustava. Egocentrizam u

dječjem mišljenju tumači se prvotnom djetetovom nesposobnošću da razlikuje sebe od okline.

Egocentrizam djeteta očituje se kao priroda njegovih misli koje ponajprije imaju zadaću

opisivanja ili usmjeravanja vlastite aktivnosti. Egocentrizam se očituje i kao nesposobnost

zauzimanja tuĎeg stajališta, tuĎeg motrišta u percepciji. Na primjer, ako se predškolskog

dječaka upita ima li njegov brat brata, odgovor će biti „ne“, jer se dječak ne moţe uţivjeti u

poloţaj svog brata, kako bi opazio svoj odnos prema njemu. Kako bi se taj dječji egocentrični

svijet prodrmao dijete mora iskusiti i stajališta drugih. Ako ne komunicira i ne interagira s

okolinom, ono neće mijenjati mentalne strukture koje su nastale iz njegovog egocentričnog

sustava, a koje mogu biti neadekvatne i voditi pogrešnom načinu mišljenja. Primjenom tog

načela na nastavni proces, proizlazi da u razredu treba ostvariti naglašenu interakciju. Mora se

ostvariti, uz interakciju izmeĎu nastavnika i učenika, i interakcija u sklopu cijelog razreda.

Preporučuje se da interakcija ima oblik koordinirane otvorene rasprave u kojoj dolazi do

razmjene informacija i ideja, do sukoba mišljenja, argumentiranih debata, te do pročišćavanja

i jasnijeg uobličavanja ideja.

Page 18: implementacija konceptualnog testa iz nuklearne fizike i fizike čestica

13

Najpoznatiji Piagetov rezultat je njegova teorija kognitivnog razvoja djeteta, počevši

od roĎenja do adolescencije. Ukupni kognitivni razvoj Piaget je podijelio na 4 faze. Svaka

faza je okarakterizirana mogućnostima mišljenja djece, ali i ograničenjima u mišljenju.

Pretpostavka je da se razvoj u svih osoba zbiva po odreĎenom redoslijedu, od manje efikasnih

do efikasnijih načina razmišljanja. Faze su redom od roĎenja djeteta:

1. Psihomotorna faza (0 – 2. godine). U tom se razdoblju najprije razvijaju refleksi i

osnovne navike. Postupno se razvija i postiţe svijest o permanentnom postojanju

materijalnih objekata; postiţe doţivljaj stalnosti predmeta: shvaća da predmeti

postoje iako nisu dostupni njegovim osjetilima. Ovaj stadij nije posebno značajan

za nastavnike fizike.

2. Predoperacijska faza (2. – 7. godine). U ovom se razdoblju dalje razvija jezik

(predstavljanje predmeta pomoću slika i riječi), različita iskustva pokušavaju se

spojiti u cjelinu. Stvara se osobni svijet, uz jako naglašeno egocentrično mišljenje,

teško mu je preuzeti tuĎe motrište. Razvija se intuitivna misao i transduktivno

mišljenje – prelazi se od jednoga pojedinačnog svojstva na drugo, a da se ne

prelazi preko općeg. Potkraj faze dijete moţe klasificirati predmete, i to samo

prema jednom obiljeţju: npr. po boji bez obzira na oblik, ili obrnuto. Dijete nema

sposobnost ureĎivanja i preureĎivanja podataka iz konkretnog iskustva ili dobivene

u eksperimentu. Nije razvijena sposobnost kauzalnog mišljenja. Dijete ne uočava

vremenski slijed uzroka i posljedice.

3. Faza konkretnih operacija (7. – 11. godine). Dijete se u ovom razdoblju ţivota

razvija iz predoperacijskog stadija prema razini u kojoj je u stanju izvoditi misaone

operacije. Većina djece s navršenih 11 godina doseţe razinu konkretnih misaonih

operacija. Postoji mogućnost logičkog zaključivanja o stvarima i dogaĎajima.

Osoba na ovoj razini razvitka u stanju je intuitivno primjenjivati jednostavne

zakone očuvanja i misliti kauzalno, ali samo ako se te mentalne operacije odnose

na sasvim konkretne objekte. Dijete moţe klasificirati predmete prema nekoliko

obiljeţja istodobno, a moţe ih poredati u niz prema jednom obiljeţju, npr. prema

veličini.

4. Faza formalnih (apstraktnih) operacija (od 11. godine nadalje). Osoba koja

dostigne ovaj stadij kognitivnog razvoja nema svih onih ograničenja koja

karakteriziraju konkretnog mislioca. Formalni mislilac je u stanju razmišljati

apstraktno i rabiti u razmišljanju propozicijsku logiku. Moţe logično razmišljati o

apstraktnim tvrdnjama i sustavno provjeravati pretpostavke. Zanima ga

pretpostavljeno i buduća dogaĎanja.

Za nastavu fizike, koja kod nas započinje u 7. razredu osnovne škole, kada se djeca

nalaze u dobi od 13 godina, najvaţnije su karakteristike posljednjih dviju faza mišljenja.

Učenici koji se u školi prvi put susreću s fizikom, nalaze se u prijelaznoj fazi razvoja od

konkretnog prema formalnom načinu mišljenja. To je vrlo vaţno za strukturiranje i izvoĎenje

nastave. Zato bi nastavni sadrţaji trebali biti predočeni na razini koja je primjerena danoj

dobi, odnosno nastavne sadrţaje treba strukturirati i razraditi tako da oni u procesu učenja

povoljno djeluju na razvoj kognitivnih struktura učenika.

Page 19: implementacija konceptualnog testa iz nuklearne fizike i fizike čestica

14

3.2 Učeničke intuitivne ideje ili pretkoncepcije - uočavanje

njihovih uloga

Svjesno iskustvo se ureĎuje kognitivnim modelima, tzv. konceptima. Stalno rastuće

iskustvo ureĎuje se u konceptualnu organizaciju tog iskustva. Sposobnost simboliziranja

omogućuje apstraktnu konceptualizaciju.

Koncepti su paketi značenja, obuhvaćaju sličnosti i razlike, uzorke, ili veze meĎu

objektima, dogaĎajima i drugim konceptima. Svaki je koncept presjecište mnoštva relacija.

Koncepti ne mogu biti ni istiniti ni laţni, nego samo konkretni ili apstraktni, sretno ili manje

sretno odabrani, korisni ili manje korisni. Zbog kompleksnosti koncepata njihovo je usvajanje

dug proces koji nema završetka. Koncepti se nikada ne razviju do konačnog, završnog oblika,

niti u svijesti osobe niti u korpusu znanja znanosti. U fizici su mnogi primjeri za to: masa,

elektron, elementarni električni naboj, itd., pa čak i neki elementarni koncepti kao što je teţina

danas nemaju konsenzualnu definiciju.

U 1970-im godinama uočeno je da učenici prije učenja odreĎenog znanstvenog

koncepta nisu tabula rasa glede tog koncepta, već o njemu imaju odreĎene ideje koje su stekli

na temelju ţivotnog iskustva i pojednostavljenog zaključivanja. Uočeno je da i u učenju fizike

veliku ulogu imaju takve učeničke intuitivne ideje ili pretkoncepcije, te da se njima mogu

tumačiti znatne poteškoće koje učenici imaju u konceptualnom razumijevanju fizike. Te

pretkoncepcije su spontano formirani koncepti koji se najčešće ne podudaraju sa znanstvenim

koncepcijama pa čine ozbiljnu prepreku u ispravnom konceptualnom usvajanju znanstvenog

znanja u školi. Od osamdesetih godina 20. stoljeća pa sve do danas, nastao je veliki broj

istraţivanja učeničkih predkoncepcija u različitim područjima fizike. Jedno takvo istraţivanje,

u području nuklearne fizike i fizike čestica, provedeno je meĎu studentima PMF-a, a rezultati

istraţivanja predstavljeni su u idućim poglavljima ovog diplomskog rada.

Učeničke koncepcije razvijaju se i mijenjaju tijekom vremena, ali su vrlo tvrdokorne i

vrlo ih je teško promijeniti. MeĎutim, iako rijeĎe, pretkoncepcije mogu biti most prema

usvajanju znanstvenih koncepata. Kako bi se to postiglo, a eliminirao njihov štetan utjecaj,

nastavni proces mora biti naglašeno interaktivan. U nastavnom procesu, prilikom interaktivne

obrade sadrţaja, prvi korak je identifikacija učeničkih pretkoncepcija, odnosno njihovo

prepoznavanje. Drugi korak je poticanje procesa konceptualne promjene u kojem učenik

odustaje od svoje pretkoncepcije i zamjenjuje ju znanstveno ispravnom koncepcijom. Taj

proces potiče nastavnik pogodno odabranim situacijama, pitanjima i argumentima, ali bitan i

aktivan sudionik tog procesa je sam učenik. U naglašeno interaktivnom ozračju učenik

odustaje od svoje pretkoncepcije tek kad mu se u raspravi predoče njemu uvjerljivi argumenti.

Proces konceptualne promjene organizira i vodi nastavnik, ali je učenik taj koji tijekom

interakcije odlučuje hoće li napustiti i rekonstruirati svoju pretkoncepciju. Rezultat ovisi o

uvjerljivosti argumenata i nije dovoljno da nastavnik ukaţe učeniku na njegove

pretkoncepcije, te da mu naprosto kaţe kojim ih znanstveno ispravnim koncepcijama mora

nadomjestiti. Sam autoritet nastavnika i znanosti nije dovoljan da učenik odustane od svojih

pretkoncepcija nego su potrebni uvjerljivi argumenti. Učenik odbacuje svoju koncepciju tek

kad se ona pokaţe nezadovoljavajućom, odnosno tek kad uvidi da je postala nefunkcionalna.

TakoĎer, da bi odbacio svoju intuitivnu ideju, nova ideja mora biti razumljiva i otpočetka

djelovati uvjerljivo, te mora djelovati plodonosnije od stare. Konačan cilj konceptualne

promjene je integracija učeničkog privatnog i školskog znanja u integralno učeničko znanje

koje je u skladu sa znanstvenim znanjem.

Page 20: implementacija konceptualnog testa iz nuklearne fizike i fizike čestica

15

3.3 Konstruktivistički pristup nastavi fizike

Konstruktivizam je kao filozofski pokret u modernom obliku nastao u drugoj polovici

20. stoljeća, a danas je prevladavajući pokret u filozofiji znanosti. Edukacijski konstruktivizam

je inačica konstruktivizma usredotočena na proces učenja i poučavanja. Edukacijski

konstruktivizam danas pokušava povezati Piagetove ideje, utjecaj učeničkih pretkoncepcija i

teoriju konceptualne promjene u jednu koherentnu sliku o problematici učenja. Temeljne

karakteristike edukacijskog konstruktivizma su da znanje nije moguće prenijeti pasivnom

primatelju i da je znanje rezultat osobne konstruktivne aktivnosti.

Sve postojeće znanje rezultat je ljudske konstrukcije. U procesu stvaranja znanja bitnu

ulogu ima konstruktivna aktivnost svijesti u stvaranju i interpretaciji iskustva. Ulazni se

podaci procesiraju i transformiraju putem slijeda kognitivnih struktura, a konačni rezultat tog

procesiranja informacija i iskustva jest znanje. Znanje i ideje nije moguće pretočiti u učeničke

glave izravnim prenošenjem, nego učenici moraju za sebe konstruirati njihovo značenje.

Još jedna bitna karakteristika edukacijskog konstruktivizma je da nastavnik

konstruktivist ne smatra da je ono što on poučava konačna i apsolutna istina. U prirodnim

znanostima moţe se reći jedino da je to najbolji način razmatranja dane situacije. Općenito,

postignuća znanosti ne znače istinu o objektivnom svijetu, i nisu apsolutna; ona su privremena

i pogrešiva, ali daju najbolji prikaz prirodnih pojava i situacija u odreĎenom vremenu.

Na temelju konstruktivističkih ideja u razredu se ostvaruje atmosfera koja maksimizira

učenje učenika. Pri izvoĎenju nastave fizike, ozračje u učionici treba biti takvo da učenicima

omogućuje konstruktivistički način učenja, tj. konstruktivno razmišljanje, osobno

konstruiranje i rekonstruiranje ideja u koordiniranoj raspravi u kojoj se razmjenjuju mišljenja

i postiţe nekakav oblik konsenzusa. Najbolji način da se to ostvari je da se novi sadrţaji daju

u obliku zanimljivih i primjerenih problemskih situacija u čijem razrješavanju aktivno

sudjeluju svi učenici. Bitno je da nastavni proces bude izrazito interaktivan. Izlaganja

nastavnika ne smiju biti dugačka, nego on treba aktivno sudjelovati u raspravi. Uloga

nastavnika je da priprema odgovarajuće problemske situacije i da koordinira raspravom.

Nastavnik mora voditi računa o onome što učenik zna, maksimizirati interakciju meţu

učenicima tako da oni mogu raspravljati, te pokusima omogućiti mnoštvo iskustava na kojima

se gradi njihovo učenje.

Nove koncepte ne treba uvoditi formalnim definicijama, bilo verbalno ili u obliku

matematičke formule, nego ih treba uvoditi operacijski, tj. kroz problemsku situaciju u kojoj

učenici tijekom rasprave uviĎaju da je u svrhu boljeg opisivanja pojave poţeljno uvesti novi

koncept. Bitno je imati na umu da učenici nisu tabula rasa, nego već imaju neko raspoloţivo

znanje koje je jedini temelj na koji mogu nadograĎivati novo znanje. Iz tog razloga, bitna

uloga nastavnika je da stekne uvid o tome kojim konceptima učenici raspolaţu. To

identificiranje učeničkih pretkoncepcija od ključne je vaţnosti za uspješno provoĎenje

procesa konceptualne promjene.

Page 21: implementacija konceptualnog testa iz nuklearne fizike i fizike čestica

16

4. Konceptualni test iz nuklearne fizike i fizike čestica

4.1 Konstruiranje konceptualnog testa

Istraţivanjem konceptualnog razumijevanja nuklearne fizike i fizike čestica, te

pojmova pomoću kojih se interpretiraju pojave i izraţavaju zakonitosti u tim područjima

fizike, dobiva se uvid o načinu razmišljanja ispitanika. Takvim istraţivanjem se dolazi do

njihovih ideja vezanih uz ta područja i općenito do ideja koje ispitanici smatraju prikladnim

za opis pojava koje primjećuju, te se dobiva uvid o iskustvu koje oni posjeduju i primjenjuju.

Proces konstrukcije konceptualnog testa temelji se na istraţivanju područja čije se

razumijevanje ţeli ispitati. Kako bi se u potpunosti ispitalo razumijevanje nekog područja

fizike potrebno je provesti opseţno istraţivanje. To nije lagan ni brz posao, jer je nuţno

osmisliti dobra pitanja i istovremeno obuhvatiti sve fundamentalne pojmove.

Nakon osmišljavanja 37 pitanja s potpitanjima (ukupno 60-ak pitanja), idući korak je

bio razgovor s ispitanicima. Ta ciljana pitanja su bila kostur razgovora, ali sam tok razgovora

nije bio striktno odreĎen i mogao se razlikovati od ispitanika do ispitanika. Na neka od pitanja

bili su ponuĎeni i neki odgovori izmeĎu kojih su se ispitanici mogli odlučiti za onaj koji

smatraju da je točan ili najbliţi točnom. Najčešće je osim odgovora bilo potrebno iznijeti i

svoja razmišljanja odnosno argumente za odabrani odgovor. Svi razgovori su voĎeni u

opuštenoj atmosferi, izrazito interaktivno, te je zato bilo moguće detaljno ispitati prirodu

pojedinih poteškoća u istraţivanom području. Razgovori su u prosjeku trajali 40-ak minuta,

snimani su i naknadno preslušavani te su se zapisivali najčešći odgovori.

Na osnovu preslušanih razgovora i dobivenih informacija, te najzastupljenijih

pogrešnih odgovora, konstruiran je konceptualni test iz nuklearne fizike i fizike čestica. Test

sadrţi 28 pitanja koja pokrivaju područja jezgara, radioaktivnosti i čestica. Pitanja su tipa

višestrukog izbora s 5 ponuĎenih odgovora od kojih je 1 odgovor točan ili najbliţi točnom.

PonuĎeni pogrešni odgovori (distraktori) predstavljaju najzastupljenije predkoncepcije, a do

njih se došlo na temelju preslušanih razgovora. Sva pitanja su konceptualnog karaktera,

ispituju razumijevanje odreĎenih pojmova i koncepata, te ne zahtijevaju nikakav oblik

računanja.

4.2 Primjena konceptualnog testa

Primjena pismenog testa, s pitanjima višestrukog izbora, korisna je u odreĎivanju

učestalosti pogrešnih ideja i miskoncepcija kod grupe ispitanika na kojoj se test primjenjuje, a

takoĎer moţe biti indikativna i za širu populaciju.

Konceptualni test iz nuklearne fizike i fizike čestica proveden je na studentima

prirodoslovno-matematičkog fakulteta. Vrijeme pisanja testa je bilo ograničeno na 45 minuta.

Rješavanju testa je pristupilo 26 studenata fizike: 17 studenata profesorskog smjera i 9

studenata istraţivačkog smjera. Svi oni posjeduju višegodišnje iskustvo učenja fizike, što u

srednjoj školi, što na prvim godinama fakulteta.

Page 22: implementacija konceptualnog testa iz nuklearne fizike i fizike čestica

17

Još treba napomenuti da bi za vjerodostojnije rezultate bilo bolje da je broj ispitanika

veći, meĎutim i zaključci dobiveni istraţivanjem na ovom manjem uzorku mogu biti

indikativni.

Testovi su ispravljani ručno, a rezultati obraĎeni kombinirano ručno i Excelom.

Analizom rezultata ţeli se dobiti uvid u opće stanje uspjeha na testu i postotak rješivosti

pojedinih pitanja, ukupno i po smjerovima.

4.3 Pregled dobivenih rezultata testiranja

Analizirajući odgovore studenata profesorskog i istraţivačkog smjera, koji su bili

testirani ovim konceptualnim testom, pokušat ćemo vidjeti koje su najčešće miskoncepcije, te

koja područja nuklearne fizike i fizike čestica su najlošije svladana i predstavljaju najveći

problem.

Na grafikonu 1 poredani su zadaci od prvog do posljednjeg te je svakom zadatku

pridruţen postotak rješivosti. Moţemo vidjeti kako 2 najuspješnije riješena zadatka imaju

85% točnih odgovora, dok je najlošiji uspjeh ostvaren u rasponu od 8% do 23% točnih

odgovora. Vrlo loš uspjeh ostvaren na pripadajućim zadacima sugerira na problematiku koju

predstavljaju studentima i zato ćemo te zadatke detaljnije analizirati te vidjeti zašto su baš oni

studentima predstavljali najveći problem.

Grafikon 1. Prikaz postotka točnih odgovora

Vidljivo je kako se postotak rješivosti razlikuje od zadatka do zadatka pa ih se, radi

bolje preglednosti i lakše analize, moţe poredati po uspješnosti rješavanja, te grafikonom 2

prikazati njihov poredak po teţini.

0

10

20

30

40

50

60

70

80

90

100

1 2 3 4 5 6 7 8 9 10 11 12 13 14 15 16 17 18 19 20 21 22 23 24 25 26 27 28

% t

očn

ih o

dgo

vora

redni broj zadatka

POSTOTAK TOČNIH ODGOVORA PO ZADACIMA

Page 23: implementacija konceptualnog testa iz nuklearne fizike i fizike čestica

18

Grafikon 2. Prikaz težine zadataka

Moţemo vidjeti kako je najmanje točnih odgovora, samo 8%, ostvareno na 23.

zadatku, te 12% točnih odgovora na 7. zadatku. Slijedi zadatak 2 (15%), zadatak 14 (19%) i

zadatak 5 sa 23% točnih odgovora. Te ćemo zadatke detaljnije analizirati, uz napomenu kako

su podebljano označeni odgovori točni odgovori.

23. Da li zakon očuvanja mase vrijedi i u mikrosvijetu i u makrosvijetu?

a) Vrijedu samo u mikrosvijetu.

b) Vrijedi samo u makrosvijetu.

c) Vrijede i u mikrosvijetu i u makrosvijetu.

d) Ne vrijedi ni u mikrosvijetu ni u makrosvijetu.

e) Masa nije stvarna veličina, pa se ne moţe definirati takav zakon.

Ovaj zadatak se pokazao kao najteţi ne samo zbog malog broja točnih odgovora, samo

8%, nego i po tome što čak 23% studenata uopće nije ponudilo nikakav odgovor. Većina

studenata koji su odgovorili je odabrala odgovor b) i to čak 55%, vjerojatno iz razloga što je u

makrosvijetu neočuvanje teţe primjetivo jer su u igri velike mase te su njihove promjene u

reakcijama zanemarive. TakoĎer, 30% studenata je odabralo odgovor c) smatrajući kako

zakon očuvanja mase uvijek vrijedi, što je moţda posljedica znanja stečenih u nastavi kemije

gdje se uči da se ukupna masa svih tvari koje sudjeluju u kemijskoj reakciji ne mijenja

tijekom kemijske reakcije. MeĎutim, zapravo se radi o očuvanju energije, a energija i masa su

povezane. Jednostavno, ako na početku imamo 1000 atoma, nakon mnogo reakcija opet ćemo

imati 1000 atoma, a masa će ostati ista ako uračunamo energiju koja se pritom izgubila i

stekla.

0

10

20

30

40

50

60

70

80

90

100

23 7 2 14 5 17 8 11 12 26 1 15 18 20 16 21 28 6 13 24 27 4 3 9 10 25 19 22

% t

očn

ih o

dgo

vora

redni broj zadatka

ZADACI POREDANI PO TEŽINI

Page 24: implementacija konceptualnog testa iz nuklearne fizike i fizike čestica

19

7. Zaokruţite točnu tvrdnju vezanu uz gustoću atoma.

a) Gustoća atoma je reda veličine svakodnevnih gustoća.

b) Gustoća atoma je otprilike 1 red veličine veća od reda veličine svakodnevnih

gustoća.

c) Gustoća atoma je 1 red veličine manja od reda veličine svakodnevnih gustoća.

d) Gustoća atoma je 3 reda veličine manja od reda veličine svakodnevnih gustoća.

e) Gustoća atoma je 5 redova veličine manja od reda veličine svakodnevnih gustoća.

Na ovom zadatku takoĎer dosta studenata (12%) nije ponudilo nikakav odgovor. Od

studenata koji su odgovorili, njih 35% je odabralo odgovor e), a 26% je odabralo odgovor d).

Dakle, oko 60% studenata smatra kako je gustoća atoma nekoliko redova veličine manja od

reda veličine svakodnevnih gustoća. Prema dobivenim odgovorima vidljivo je kako je

zanemarena činjenica da je atom osnovna gradivna jedinica svakog tijela, odnosno da se sve

sastoji od različitih atoma – od metala preko ţivih bića do vode i zraka. TakoĎer, otprilike

petina studenata smatra kako je gustoća atoma nešto veća od reda veličine svakodnevnih

gustoća, što je zaista iznenaĎujuće velik postotak i bilo bi zanimljivo čuti njihovo

obrazloţenje.

2. Da li su svi atomi jednako gusti?

a) Gustoća atoma s većim rednim brojem je manja zbog većih sila odbijanja protona.

b) Svi su atomi jednako gusti jer su većinom prazan prostor.

c) Atomi s većim rednim brojem su gušći jer masa atoma raste linearno s

rednim brojem, a volumen pada.

d) Ne, jer gustoća atoma raste s ukupnim brojem nukleona sadrţanih u atomskoj

jezgri.

e) Niti jedan odgovor nije točan.

Najviše studenata (33%) je odabralo odgovor b). Razlog tomu moţda treba traţiti u

skali na kojoj su studenti gledali. Drugi razlog bi mogao biti u povlačenju paralele s 1.

zadatkom, gdje je otprilike polovica studenata točno odgovorila da su sve jezgre (gotovo)

jednako guste, pa su i ovdje primjenjivali sličan recept. Svi ostali odgovori su bili potpuno

jednako zastupljeni što moţe ukazivati na pogaĎanje rješenja.

14. Radioaktivni element ima vrijeme poluraspada 10 minuta. U tom vremenskom intervalu

se neke od jezgara elementa ne raspadnu. Kolika je vjerojatnost da se u idućih 10 minuta

raspadnu i te jezgre?

a) Manja od 50%.

b) Veća od 50%.

c) Točno 50%.

d) Treba nacrtati graf eksponencijalne funkcije i očitati.

e) Ne moţe se odrediti.

Page 25: implementacija konceptualnog testa iz nuklearne fizike i fizike čestica

20

Samo je 19% studenata točno odgovorilo da se vjerojatnost raspada jezgara ne mijenja.

Gotovo polovica (46%) studenata smatra kako je vjerojatnost, nakon što proĎe jedno vrijeme

poluraspada, zasigurno manja od 50%. Bilo bi zanimljivo čuti mišljenja za koliko misle da je

manja, ali vjerojatno smatraju da to slijedi eksponencijalni zakon. Zanimljivo je da 15%

studenata misli da se to ne moţe odrediti, a 12% studenata (ni jedan od njih nije s

istraţivačkog smjera) smatra da se vjerojatnost povećava, valjda smatrajući da se jezgra

zasigurno mora raspasti u idućem vremenskom intervalu jednakom vremenu poluraspada.

Kako god bilo, očito je da većina studenata nije usvojila da radioaktivnost ne ovisi o starosti

jezgre, odnosno da starije jezgre „umiru“ jednakom brzinom kao i mlaĎe, tj. jednaka je

vjerojatnost da se raspadne vrlo mlada jezgra, koja je roĎena primjerice prije samo 1 dan, kao

i vrlo stara, primjerice roĎena prije milijun godina.

5. Da li se sastavnice jezgre raspadaju bez pobude?

a) I proton i neutron se raspadaju nakon dovoljno vremena, bez obzira da li su u

jezgri ili ne.

b) Proton se raspada kad je u vezanom stanju, a neutron kad je slobodan.

c) Neutron se raspada kad je u vezanom stanju, a proton kad je slobodan.

d) Ni proton ni neutron se ne raspadaju u vezanom stanju.

e) Ni proton ni neutron se ne raspadaju u slobodnom stanju.

Ovo je jedan od zadataka kod kojeg se značajno razlikuju odgovori studenata

profesorskog i odgovori studenata istraţivačkog smjera:

Grafikon 3. Uspješnost profesora – zd. 5 Grafikon 4. Uspješnost istraživača – zd. 5

Kod studenata profesorskog smjera najzastupljeniji je odgovor a) s 38%, zatim točan

odgovor d) s 30%, te odgovori b) i e) s 13%. Uočljivo je kako 57% studenata istraţivačkog

smjera (koji su odgovorili a), b) ili c)), pogrešno smatraju da se proton raspada, dok 13%

studenata smatra, takoĎer pogrešno, da se neutron ne raspada u slobodnom stanju.

Kod studenata istraţivačkog smjera zastupljena su samo 2 odgovora. Samo 12% ih je

točno odgovorilo, dok ih čak 88% smatra da se proton raspada kad je u vezanom stanju, očito

neznajući da ne postoji lakši barion u koji bi se mogao raspasti.

0

20

40

60

80

100

a) b) c) d) e)

38%

13%6%

30%

13%

0

20

40

60

80

100

a) b) c) d) e)

0%

88%

0%

12%

0%

Page 26: implementacija konceptualnog testa iz nuklearne fizike i fizike čestica

21

Najuspješnije riješeni zadaci su 19. i 22. zadatak s 85% točnih odgovora, te 3., 9., 10. i

25. zadatak sa 74% točnih odgovora. Analizirati ćemo detaljnije i te zadatke.

19. Zašto se prilikom raspada jezgre produkti raziĎu sa velikom kinetičkom energijom?

a) Jer ukupna energija prilikom raspada nije očuvana.

b) Jer masa pojedinih nukleona nije očuvana.

c) Ta energija je došla od energije vezanja.

d) Ta energija je došla od energije mirovanja.

e) Ne raziĎu se s velikom kinetičkom energijom ako sama jezgra prije raspada miruje

Uz odličan postotak točnih odgovora (85%), jedini zastupljeni pogrešan odgovor je bio

d), što pokazuje da studenti primjenjuju zakon očuvanja energije i jasno im je odakle

produktima kinetička energija, odnosno jasan im je sam pojam energije vezanja.

22. Ako bi postojao dio Svemira koji se sastoji od antičestica, kako bi tamo izgledao vodikov

atom i kako bi se ponašao?

a) Izgledao bi jednako kao i vodikov atom i imao bi ista svojstva.

b) Izgledao bi jednako kao i vodikov atom, ali ne bi imao ista svojstva.

c) Sastojao bi se od antiprotona i pozitrona, i imao bi ista svojstva.

d) Sastojao bi se od antiprotona i pozitrona, i ne bi imao ista svojstva.

e) Nemoguće je sastaviti takav „anti“-atom.

Studenti su s 85% točnih odgovora pokazali da im je jasan koncept materije i

antimaterije.

3. Zašto nuklearne mase nisu jednostavni višekratnici mase nukleona?

a) Zbog energije vezanja jezgre.

b) Zbog uračunate mase elektrona.

c) Zato što su proton i neutron bitno različitih masa.

d) Zato što mase protona i neutrona nisu cjelobrojne vrijednosti.

e) Zato što se jezgre vrte, tj. imaju spin.

Studenti su još jednom pokazali kako dobro (73% točnih odgovora) vladaju pojmom

energija vezanja. To se posebno odnosi na studente istraţivačkog smjera koji su svi odabrali

točan odgovor. Studenti profesorskog smjera su odabirali točan odgovor u 59% slučajeva, a u

24% slučajeva odabiran je odgovor c), jer su smatrali da to ima veze s s tim što su proton i

neutron bitno različitih masa.

Page 27: implementacija konceptualnog testa iz nuklearne fizike i fizike čestica

22

9. Da li je moguće da elektron vezan uz jezgru s puno naboja u nju uĎe?

a) Nije moguće. Elektron ima samo dozvoljena stanja van jezgre.

b) Nije moguće zbog jakih nuklearnih sila koje odbijaju elektron.

c) Nije moguće. Elektron se moţe nalaziti samo oko jezgre, kako god blizu ili daleko.

d) Moguće je. Elektron se može naći u jezgri iako je vjerojatnost za to mala.

e) Moguće je zbog jakih nuklearnih sila koje privlače elektron.

Na ovom pitanju su studenti istraţivačkog smjera ponovno ostvarili stopostotan

učinak, dok su studenti profesorskog smjera odabirali točan odgovor ponovno u 59%

slučajeva.

Grafikon 5. Uspješnost profesora – zd. 9

Odabirući odgovore a) i c), vidimo da čak 35% studenata profesorskog smjera

pogrešno smatra da se elektron ne moţe naći u jezgri, dok studenti istraţivačkog smjera znaju

da je elektron opisan s oblakom vjerojatnosti koji se proteţe od centra djelovanja sile, koji se

nalazi u jezgri, pa do beskonačnosti. Stoga, najveća vjerojatnost jest da se elektron nalazi na

udaljenosti polumjera atoma, ali ipak postoji mala vjerojatnost da se on naĎe u jezgri.

10. Kako to da su jezgre stabilne ako se protoni elektrostatski odbijaju?

a) Elektrostatska Coulombova sila ne djeluje umutar jezgre.

b) Elektrostatska sila je uravnotežena nuklearnom silom privlačenja.

c) Jezgra sadrţi jednak broj elektrona i protona pa se njihovo meĎudjelovanje

neutralizira.

d) Neutroni u jezgri sprječavaju izravno meĎudjelovanje protona jednih na druge.

e) Moguće je ako se neutroni postave točno izmeĎu protona kako bi neutralizirali

njihovu silu odbijanja.

73% studenata zna da nuklearna sila privlačenja uravnoteţuje elektrostatsku silu

odbijanja meĎu protonima, tako da je ukupna sila na svaki nukleon u jezgri jednaka nula. 23%

studenata smatra da sami neutroni u jezgri igraju značajnu ulogu (odgovor d)). Istina, neutroni

imaju značajnu ulogu, ali ne sprječavaju meĎudjelovanje protona, nego je s neutronima lakše

uravnoteţiti jezgru. Dodavanjem neutrona u jezgru dodaje se nuklearna komponenta sile, dok

se dodavanjem protona dodaje komponenta i nuklearne i elektrostatske sile.

0

10

20

30

40

50

60

a) b) c) d) e)

17,65%

0%

17,65%

58,82%

5,88%

Page 28: implementacija konceptualnog testa iz nuklearne fizike i fizike čestica

23

25. Koja od fundamentalnih sila je zasluţna za trenje?

a) Gravitacijska.

b) Elektromagnetska.

c) Jaka.

d) Slaba.

e) Niti jedna, jer je trenje nekonzervativna sila.

Ovo je bilo još jedno pitanje u kojem su studenti istraţivačkog smjera pokazali bolje

znanje od studenata profesorskog smjera.

Grafikon 6. Uspješnost profesora – zd. 25 Grafikon 7. Uspješnost istraživača – zd. 25

Studenati profesorskog smjera su u 24% slučajeva zaokruţivali a) kao točan odgovor,

smatrajući kako trenje spada u gravitacijsku silu, iako sve kontaktne sile spadaju u

elektromagnetski spektar sila.

Neka od zanimljivijih pitanja:

1. Da li su sve jezgre jednako guste?

a) Gustoća većih jezgri je manja zbog većih sila odbijanja protona.

b) Sve su jezgre jednako guste zbog kratkog dosega sile koja ih drži na okupu.

c) Veće jezgre su gušće zbog većeg broja čestica na relativno istom prostoru.

d) Ne, jer gustoća jezgre raste s omjerom broja protona i neutrona u jezgri.

e) Ne, jer gustoća jezgre pada s omjerom broja protona i neutrona u jezgri.

Slično pitanje se nalazilo u konceptualnom istraţivanju koje ju 2010.g. provedeno

meĎu studentima nuklearne fizike u St. Andrewsu i Edinburghu.

Što se tiče našeg istraţivanja, ovo je jedno od rijetkih pitanja na kojem su studenti

profesorskog smjera ostvarili značajno bolji uspjeh od studenata istraţivačkog smjera:

0

20

40

60

80

100

a) b) c) d) e) ništa

24%

65%

6%0% 0%

6%

0

20

40

60

80

100

a) b) c) d) e)

0%

89%

0% 0%11%

Page 29: implementacija konceptualnog testa iz nuklearne fizike i fizike čestica

24

Grafikon 8. Uspješnost studenata oba smjera – zd. 1

Studenti profesorskog smjera su odgovarali na ovo pitanje s 53% uspješnosti, dok su

studenti istraţivačkog smjera ostvarili uspjeh od 33%. Ukupno gledajući, čak 54% studenata

smatra da nisu sve jezgre jednako guste. Studenti istraţivačkog smjera su se najčešće

odlučivali za odgovor a) oslanjajući se na to da veća jezgra ima više protona, te da je zbog

toga veća sila odbijanja protona, a samim time i manja gustoća takvih jezgri. Studenti očito

nisu imali na umu da osim odbojne elektrostatske sile djeluje i privlačna sila kratkog dosega.

Što se tiče škotskog istraţivanja, znatan udio studenata (preko 50%) je tvrdio da

gustoća raste s nukleonskim brojem. Glavni razlog koji su studenti navodili je da „više

nukleona znači tjesnije stanje“, odnosno „povećanje masenog broja znači da je jezgra sve

gušća i gušća“.

11. Da li radioaktivni izotop moţe emitirati samo jedan neutron?

a) Ne moţe, jer neutron nije ni α, ni β, ni γ zračenje.

b) Ne moţe, jer se time narušavaju neki zakoni očuvanja.

c) Ne moţe, jer to nikad nije energetski povoljan proces.

d) Može, iako se to rijetko dešava u odnosu na ostale tipove raspada.

e) Moţe, i to se dešava češće od ostalih tipova raspada.

Značajna je razlika u odgovorima studenata različitih smjerova:

Grafikon 9. Uspješnost studenata oba smjera – zd. 11

0

10

20

30

40

50

60

a) b) c) d) e) ništa

11,76%

52,94%

11,76%

23,53%

44,4%

33,3%

11,1%11,1%

profesori

istraživači

0

10

20

30

40

50

60

70

a) b) c) d) e) ništa

18%

35%

18%23%

6%

22%

67%

11%

profesori

istraživači

Page 30: implementacija konceptualnog testa iz nuklearne fizike i fizike čestica

25

Točno je odgovorilo 67% studenata istraţivačkog smjera, a samo 23% studenata

profesorskog smjera. Najzastupljeniji pogrešan odgovor (ujedno i općenito najzastupljeniji

odgovor kod studenata profesorskog smjera) je odgovor b). Bilo bi zanimljivo čuti za koje

zakone očuvanja smatraju da nisu očuvani.

12. Da li na radioaktivnost nekog elementa utječe malo povišenje njegove temperature?

a) Ne, ni u kakvoj mjeri.

b) Da, zato što sigurno na apsolutnoj nuli nema radioaktivnosti.

c) Da, izotopi više temperature puno jače zrače.

d) Da, izotopi više temperature puno slabije zrače.

e) Da, temperatura utječe na radioaktivnost, ali iznimno malo.

42% studenata ispravno smatra da temperatura, iako iznimno malo, utječe na

radioaktivnost. Pribliţno isti postotak studenata (38%) smatra da povišenje temperature

elementa ni u kakvoj mjeri ne utječe na radioaktivnost.

Grafikon 10. Uspješnost studenata – zd. 12

Ne treba zanemariti ni da se za odgovor c) odlučilo 15% studenata, od čega niti jedan s

istraţivačkog smjera, što znači da gotovo svaki četvrti student profesorskog smjera (24%)

pogrešno smatra da izotopi više temperature puno jače zrače.

13. Zamislite sustav od 1000 radioaktivnih atoma. Koliko vremena poluţivota je prošlo kad se

raspalo 750 atoma?

a) Pola vremena poluţivota.

b) Jedno i pol vrijeme poluţivota.

c) Dva vremena poluživota.

d) IzmeĎu dva i tri vremena poluţivota.

e) Četiri vremena poluţivota.

Isto ovo pitanje je postavljeno u škotskom konceptualnom istraţivanju. Razlika je samo što

su u tom testu bili ponuĎeni slijedeći odgovori:

0

10

20

30

40

50

a) b) c) d) e)

39%

4%

15%

0%

42%

Page 31: implementacija konceptualnog testa iz nuklearne fizike i fizike čestica

26

A) 0.25 B) 0.41 C) 0.50 D) 0.75

E) 1.3 F) 2.0 G) 2.4 H) 4.0

Što se tiče našeg istraţivanja, na ovom pitanju su studenti istraţivačkog smjera još

jednom postigli značajno bolji rezultat.

Grafikon 11. Uspješnost studenata oba smjera – zd. 13

I dok je njih 78% točno odgovorilo da se radi o dva vremena poluţivota, čak 53%

studenata profesorskog smjera smatra da je prošlo jedno i pol vrijeme poluţivota. Očito

studenti profesorskog smjera nisu dobro usvojili pojam vremena poluraspada (poluţivota).

Što se tiče škotskog istraţivanja, studenti koji su pogrešno odgovarali najčešće su

zaokruţivali odgovor C) 0.50, vjerojatno smatrajući da se broj čestica raspada linearno, a ne

eksponencijalno; ako se 500 atoma raspadne nakon 1 vremena poluţivota, onda se 250

raspadne nakon 0.5 vremena poluţivota. TakoĎer je u pogrešan odgovor uključena i zbrka oko

broja jezgara koje su se raspale i broja preostalih jezgara. Slijedeći najčešći pogrešan odgovor

je bio B) 0.41; studenti su koristili dobru formulu, ali je opet zabuna oko broja raspadnutih i

broja preostalih jezgri. Uočljivo je da od studenata koji su dali točan odgovor, velika većina

nije koristila formulu nego je prosudila prema definiciji vremena poluraspada, dok je većina

studenata koji su zaokruţili netočan odgovor koristila matematičku formulu.

16. Jedan radioaktivni atom ima vrijeme poluţivota 10 minuta. Ako se u tom vremenskom

intervalu atom ne raspadne, koja je od slijedećih tvrdnji istinita?

a) Nakon isteka vremena poluživota, vjerojatnost raspada ostaje

nepromijenjena.

b) Nakon isteka vremena poluţivota smanjuje se vjerojatnost raspada.

c) Nakon isteka vremena poluţivota povećava se vjerojatnost raspada.

d) Jedan atom će se sigurno raspasti nakon dva vremena poluţivota.

e) Jedan izdvojeni atom se nikada neće raspasti.

Tek 50% studenata, dakle svaki drugi student, zna da vjerojatnost raspada nakon isteka

vremena poluţivota ostaje nepromijenjena.

0

10

20

30

40

50

60

70

80

a) b) c) d) e) ništa

53% 47%

11%

78%

11%

profesori

istraživači

Page 32: implementacija konceptualnog testa iz nuklearne fizike i fizike čestica

27

Grafikon 12. Uspješnost studenata – zd. 16

Otprilike svaki treći student misli da je točan odgovor c), smatrajući da se vjerojatnost

raspada povećava jer je jezgra starija, dok svaki deveti student smatra da se vjerojatnost

raspada nakon isteka vremena poluţivota smanjuje.

18 Da li se jezgra moţe u jednom trenu (ne u lancu) raspasti na više od dvije čestice

odjednom?

a) Može, to je prirodna stvar.

b) Moţe, ali samo u nuklearnim reaktorima.

c) Ne moţe, jer je energetski uvijek isplativije raspasti se na dva dijela.

d) Ne moţe zbog nekih zakona očuvanja.

e) Niti jedan od odgovora nije točan.

Ponovno su studenti istraţivačkog smjera ostvarili odličan učinak s 89% točnih

odgovora. Nasuprot njima, samo 24% studenata profesorskog smjera smatra da je prirodna

stvar da se jezgra odjednom raspadne na više od dvije čestice.

Grafikon 13. Uspješnost studenata oba smjera – zd. 18

Više od polovice studenata (53%) profesorskog smjera misli da se jezgra ne moţe

odjednom raspasti na više od dvije čestice, što zbog toga jer je energetski neisplativo (35%),

što zbog nekih zakona očuvanja (18%).

0

10

20

30

40

50

a) b) c) d) e)

50%

11%

35%

4%0%

0

10

20

30

40

50

60

70

80

90

a) b) c) d) e) ništa

23%

12%

35%

18%12%

89%

11%

profesori

istraživači

Page 33: implementacija konceptualnog testa iz nuklearne fizike i fizike čestica

28

20. Foton valne duljine 500 nm se pusti u gibanje od točke 1 do točke 2 kao na slici. Koji

crteţ najbolje odgovara putanji fotona?

46% studenata je ispravno odgovorilo da je putanja fotona ravna linija, dok se velikih

42% odlučilo za odgovor a) smatrajući da je putanja fotona valovita linija. Razlog tome bi

mogao biti što se često crta valovita linija kad se skicira gibanje fotona.

21. Da li čestice bez naboja (npr. neutron) imaju antičestice?

a) Da, svaka čestica ima antičesticu.

b) Ne, jer nemaju naboj, pa nema što biti „anti“.

c) Da, jer iako nemaju naboj, sve su čestice sastavljene od manjih naboja.

d) Ne, jer samo elektroni i protoni imaju antičestice.

e) Ne, zato što antičestice ne postoje.

Grafikon 14. Uspješnost studenata oba smjera – zd. 21

Iako su studenti istraţivačkog smjera ravnopravno zaokruţivali i odgovor a) i odgovor

c), treba istaknuti da ih čak 88% ispravno smatra da svaka čestica ima antičesticu. To je slučaj

i kod 76% studenata profesorskog smjera. MeĎutim, iz raspodjele odgovora vidi se kako baš i

nije poznato što je kod antičestica zapravo „anti“.

0

10

20

30

40

50

60

a) b) c) d) e) ništa

52,9%

17,6%23,5%

5,9%

44,4% 44,4%

11,1%

profesori

istraživači

Page 34: implementacija konceptualnog testa iz nuklearne fizike i fizike čestica

29

24. Za što su zasluţne jaka i slaba sila?

a) Jaka sila drţi jezgru na okupu, a slaba nukleone.

b) Slaba sila drţi jezgru na okupu, a jaka nukleone.

c) Jaka sila drži na okupu i jezgru i nukleone, a slaba je zaslužna za nešto drugo.

d) Slaba sila drţi na okupu i jezgru i nukleone, a jaka je zasluţna za nešto drugo.

e) Niti jedan od odgovora nije točan.

Svi studenti istraţivačkog smjera su točno odgovorili za što su zasluţne jaka i slaba

sila, dok dvije trećine studenata profesorskog smjera to ne zna.

Grafikon 15. Uspješnost studenata oba smjera – zd. 24

26. Da li sila djeluje trenutno ili joj treba neko vrijeme?

a) Sila djeluje trenutno.

b) Signal putuje brzinom većom od brzine svjetlosti, ali sila ne djeluje trenutno.

c) Signal putuje brzinom jednakoj brzini svjetlosti.

d) Signal putuje brzinom jednakoj ili manjoj od brzine svjetlosti.

e) Niti jedan od odgovora nije točan.

Grafikon 16. Uspješnost studenata oba smjera – zd. 26

I dok 78% studenata istraţivačkog smjera ispravno smatra da signal putuje brzinom

jednakoj ili manjoj od brzine svjetlosti, ovisno o prijenosniku sile, čak 71% studenata

profesorskog smjera pogrešno smatra kako sila djeluje trenutno.

0

20

40

60

80

100

a) b) c) d) e)

35%24%

35%

6%

100%

profesori

istraživači

0

10

20

30

40

50

60

70

80

a) b) c) d) e)

71%

6%

23%

11%

78%

11%

profesori

istraživači

Page 35: implementacija konceptualnog testa iz nuklearne fizike i fizike čestica

30

4.4 Usporedba rezultata testiranja

Testirani uzorak se sastojao od studenata fizike profesorskog smjera i studenata fizike

istraţivačkog smjera. Broj ispitanika nije bio jako velik, pogotovo studenata istraţivačkog

smjera (9 studenata), meĎutim i na takvom manjem broju ispitanika mogu se uočiti neke

razlike u razumijevanju ovog područja fizike.

Obradom rezultata koje su u provedenom testiranju ostvarili studenti oba smjera

dobije se grafikon 17 na kojem moţemo vidjeti raspodjelu smjerova po uspjehu.

Grafikon 17. Raspodjela studenata oba smjera po uspjehu ostvarenom na testu

S obzirom na to da nije bio jednak broj studenata po smjeru, radi bolje preglednosti se

na ordinati nalazi broj ispitanika čija rješenja pripadaju odreĎenom postotnom razredu

podijeljen s ukupnim brojem članova u toj skupini.

Prvo što se vidi iz grafa je stupanj teţine koji je ovaj konceptualni test predstavljao

studentima jednog i drugog smjera. Najveći udio studenata profesorskog smjera je ostvario

rezultat u rasponu od 31% do 40%, a studenata istraţivačkog smjera u rasponu od 61% do

70%. I dok postoje studenti profesorskog smjera koji su točno riješili manje od 30% testa

(najslabije ostvaren uspjeh je 14%), najlošiji ostvaren rezultat studenata istraţivačkog smjera

je 39%. Treba istaknuti da je samo manje od polovice studenata profesorskog smjera (47%)

ostvarilo bolji uspjeh od najneuspješnijeg studenta istraţivačkog smjera. Najuspješniji

studenti profesorskog smjera su imali 64% točnih odgovora, ali od njih je bolji uspjeh

ostvarilo čak trećina studenata istraţivačkog smjera, a najbolji od njih je imao 82% točno

riješenog testa. I više je nego očito kako postoji velika razlika u znanju i razumijevanju ovog

područja fizike kod studenata različitih smjerova. Osnovni razlog tomu je razlika u kolegijima

koji se slušaju na različitim smjerovima.

0

0,05

0,1

0,15

0,2

0,25

0,3

0,35

N/u

kup

no

postotni razred

RASPODJELA PO USPJEHU

profesorski smjer

istraživački smjer

Page 36: implementacija konceptualnog testa iz nuklearne fizike i fizike čestica

31

Na grafikonu 18 poredani su zadaci od prvog do posljednjeg te je svakom zadatku

pridruţen postotak rješivosti, posebno za svaki od smjerova.

Grafikon 18. Prikaz postotka točnih odgovora po smjerovima

Iz grafa se moţe prebrojati kako su na čak tri četvrtine pitanja studenti istraţivačkog

smjera ostvarili bolji uspjeh od studenata profesorskog smjera. Prvih 10 pitanja su iz područja

jezgara, zatim 9 pitanja iz radioaktivnosti, te 9 pitanja iz čestica. Na deset pitanja o jezgrama

studenti oba smjera su odgovarali otprilike s jednakim uspjehom (šest prema četiri u korist

studenata istraţivačkog smjera), meĎutim ako se pogledaju pitanja o radioaktivnosti vidi se da

su studenti istraţivačkog smjera s boljim uspjehom odgovorili na čak sedam od devet pitanja.

Razlika je još veća što se tiče čestica, gdje su studenti profesorskog smjera ostvarili bolji

uspjeh na samo jednom od devet pitanja. Ipak, tolika razlika u uspjehu nije tako začuĎujuća

ako znamo da na istraţivačkom smjeru postoje zasebni kolegiji Nuklearna fizika i Fizika

elementarnih čestica, dok na profesorskom smjeru postoji samo kolegij Odabrana poglavlja

nuklearne fizike i fizike čestica.

Na 12 pitanja (43%) točno je odgovorilo preko 75% studenata istraţivačkog smjera,

dok je tako velik postotak točnih odgovora studentima profesorskog smjera pošao za rukom

na samo 2 pitanja. Što se tiče pitanja koja su se pokazala teţima, ona su najčešće bila

problematična za obje skupine ispitanika iz čega se moţe vidjeti kako studenti oba smjera

imaju konceptualnih problema s istim područjima, ali ipak studenti istraţivačkog smjera u

pravilu i na svakom od tih zadataka ostvaruju bolji uspjeh.

0

20

40

60

80

100

1 2 3 4 5 6 7 8 9 10 11 12 13 14 15 16 17 18 19 20 21 22 23 24 25 26 27 28

% t

očn

ih o

dgo

vora

redni broj zadatka

POSTOTAK TOČNIH ODGOVORA PO ZADACIMA I SMJEROVIMA

profesori

istraživači

Page 37: implementacija konceptualnog testa iz nuklearne fizike i fizike čestica

32

Zaključak

Ovaj diplomski rad posvećen je istraţivanju konceptualnog razumijevanja nuklearne

fizike i fizike čestica. Konceptualni testovi iz ovog područja nisu tako brojni i ne provode se

često poput, na primjer, konceptualnih testova iz mehanike, te je iz tog razloga još

zanimljivije vidjeti ostvarena postignuća.

Koncept energije vezanja se pokazao kao najbolje usvojen koncept. Studenti su

usvojili da, zbog sadrţane negativne energije vezanja, nuklearne mase nisu jednostavni

višekratnici mase nukleona, kao i da je masa vezanog sustava manja od mase mirovanja

pojedinih komponenti. TakoĎer su usvojili da se prilikom raspada jezgre produkti raziĎu s

velikom kinetičkom energijom baš zahvaljujući energiji vezanja.

Najviše problema studenti imaju s konceptom gustoće atoma. Ne samo da su

podijeljena mišljenja da li gustoća atoma s većim rednim brojem pada, raste ili se ne mijenja,

nego i smatraju da gustoća atoma nije usporediva s gustoćom sve materije koja nas okruţuje.

Kod radioaktivnosti, najveći problem čini osnovni koncept – vrijeme poluraspada.

Iako su prilikom intervjuiranja gotovo svi studenti znali iskazati riječima što to jest, na testu

se pokazalo kako imaju problema s primjenom. Očito je kako studenti nisu usvojili da je

jednaka vjerojatnost da se raspadne vrlo mlada ili vrlo stara jezgra. TakoĎer, kada je u pitanju

samo jedan radioaktivni atom, kod studenata prevladava mišljenje da ako se nakon jednog

vremena poluţivota atom ne raspadne vjerojatnost raspada se povećava, a znatan udio

studenata čak smatra da će se atom sigurno raspasti nakon dva vremena poluţivota.

Posebno zanimljiv koncept su antičestice. Vidljivo je da se studentima urezala u

pamćenje rečenica koja se često čuje na predavanjima, kako svaka čestica ima svoju

antičesticu, ali ipak postoji problem u shvaćanju zašto to vrijedi i za čestice bez naboja.

I još jedna izrazito interesantna dvojba je da li sila djeluje trenutno ili joj treba neko

vrijeme. Tek svaki četvrti student ispravno smatra da signal sile putuje brzinom jednakoj ili

manjoj od brzine svjetlosti, ovisno o prijenosniku sile.

Općenito, ostvareni uspjeh je moţda nešto ispod očekivanoga, osobito ako imamo na

umu da su studenti fizičkog odsjeka već usmjereni u područje prirodnih znanosti, tehnike i

tehnologije, te da već imaju višegodišnje iskustvo učenja fizike. Posebice se vidi prostor za

napredak kod studenata profesorskog smjera koji bi trebali biti temelj budućeg poučavanja

konceptualnom razumijevanju.

Konceptualno razumijevanje se smatra najvaţnijom razinom postignuća u učenju

fizike, jer je ono nuţno za ostvarivanje viših kognitivnih razina postignuća. Iz tog razloga

postoji veliki interes za unapreĎivanjem konceptualnog razumijevanja fizike na svim

razinama školstva, a konceptualni test iz nuklearne fizike i fizike elementarnih čestica, koji je

temelj ovog diplomskog rada, mogao bi pomoći u unaprjeĎenju konceptualnog razumijevanja

tog područja.

Page 38: implementacija konceptualnog testa iz nuklearne fizike i fizike čestica

33

Literatura:

Andrilović, V. i Čudina-Obradović M. (1985). Osnove opće i razvojne psihologije, Tiskara,

Nova Gradiška

Dadić, Ţ. (1992). Povijest ideja i metoda u matematici i fizici, Školska knjiga, Zagreb

Kohnle, A., Mclean, S., and Aliotta, M. (2011), Towards a conceptual diagnostic survey in

nuclear physics, IOP, European Journal of Physics 32

Krsnik, R. (2008). Suvremene ideje u metodici nastave fizike, Školska knjiga, Zagreb

Pećina, P., ur. (2009). Zbornik radova. Interaktivna nastava fizike., Hrvatsko fizikalno

društvo, Zagreb

Rutar, A. (2007) Uloga brzih testova u nastavnom procesu. Diplomski rad. Zagreb: PMF

http://ahyco.ffri.hr/povijestfizike/

http://eskola.hfd.hr/

Page 39: implementacija konceptualnog testa iz nuklearne fizike i fizike čestica

34

Reference:

Pećina, P., ur. (2009) Veliki hadronski sudarivač: zašto mu se divimo i zašto od njega

strepimo? Zagreb: Hrvatsko fizikalno društvo (str. u tekstu 1)

Dadić, Ţ. (1992) Povijest ideja i metoda u matematici i fizici, str. 11., Zagreb: Školska knjiga

(str. u tekstu 2)

http://ahyco.ffri.hr/povijestfizike/stari_filozofija.htm (str. u tekstu: 3)

http://ahyco.ffri.hr/povijestfizike/srednji_uvod.htm (str. u tekstu: 4)

http://ahyco.ffri.hr/povijestfizike/novi_uvod.htm (str u tekstu: 5)

Dadić, Ţ., (1992) Povijest ideja i metoda u matematici i fizici, str. 178., Zagreb: Školska

knjiga (str. u tekstu: 6)

http://eskola.hfd.hr/nazivlje/leksikon_fizike.htm (str. u tekstu: 7)

Rutar, A. (2007) Uloga brzih testova u nastavnom procesu. Diplomski rad. Zagreb: PMF (str.

u tekstu: 8)

Krsnik, R., Suvremene ideje u metodici nastave fizike, str 12. (str. u tekstu: 8)

Andrilović, V. i Čudina-Obradović M., Osnove opće i razvojne psihologije, str. 155. (str. u

tekstu: 8)

Pećina, P., ur. (2009) Interaktivna nastava fizike – utemeljenje, iskustva, perspektive. Zagreb:

Hrvatsko fizikalno društvo (str. u tekstu: 12)

Kohnle, A., Mclean, S., and Aliotta, M. (2011), Towards a conceptual diagnostic survey in

nuclear physics (str. u tekstu: 23 i 26)

Page 40: implementacija konceptualnog testa iz nuklearne fizike i fizike čestica

35

Prilog

Konceptualni test iz nuklearne fizike i fizike

elementarnih čestica

Na svako je pitanje jedan odgovor točan ili najbliţi točnom.

Vrijeme pisanja testa je 45 minuta.

1. Da li su sve jezgre jednako guste?

a) Gustoća većih jezgri je manja zbog većih sila odbijanja protona.

b) Sve su jezgre jednako guste zbog kratkog dosega sile koja ih drţi na okupu.

c) Veće jezgre su gušće zbog većeg broja čestica na relativno istom prostoru.

d) Ne, jer gustoća jezgre raste s omjerom broja protona i neutrona u jezgri.

e) Ne, jer gustoća jezgre pada s omjerom broja protona i neutrona u jezgri.

2. Da li su svi atomi jednako gusti?

a) Gustoća atoma s većim rednim brojem je manja zbog većih sila odbijanja protona.

b) Svi atomi su jednako gusti jer su većinom prazan prostor.

c) Atomi s većim rednim brojem su gušći jer masa atoma raste linearno s rednim brojem, a

volumen pada.

d) Ne, jer gustoća atoma raste s ukupnim brojem nukleona sadrţanih u atomskoj jezgri.

e) Niti jedan odgovor nije točan.

3. Zašto nuklearne mase nisu jednostavni višekratnici mase nukleona?

a) Zbog energije vezanja jezgre.

b) Zbog uračunate mase elektrona.

c) Zato što su proton i neutron bitno različitih masa.

d) Zato što mase protona i neutrona nisu cjelobrojne vrijednosti

e) Zato što se jezgre vrte, tj. imaju spin.

4. Kakva je masa vezanog sustava ( A + B ) u odnosu na masu mirovanja pojedinih komponenti

A i B?

a) Manja od mase zbroja pojedinih komponenti A i B, zbog sadrţane negativne energije

vezanja.

b) Veća od mase zbroja pojedinih komponenti A i B, zbog sadrţane pozitivne energije

vezanja.

c) Jednaka masi zbroja pojedinih komponenti A i B, jer masa mora ostati očuvana.

d) Veća od mase zbroja pojedinih komponenti A i B, zbog sadrţane energije vezanja koja je

ustvari masa.

e) Veća od mase zbroja pojedinih komponenti A i B, jer se u vezanom sustavu nalaze

dodatne vezajuće komponente.

Page 41: implementacija konceptualnog testa iz nuklearne fizike i fizike čestica

36

5. Da li se sastavnice jezgre raspadaju?

a) I proton i neutron se raspadaju nakon dovoljno vremena, bez obzira da li su u jezgri ili ne.

b) Proton se raspada kad je u vezanom stanju, a neutron kad je slobodan.

c) Neutron se raspada kad je u vezanom stanju, a proton kad je slobodan.

d) Ni proton ni neutron se ne raspadaju u vezanom stanju.

e) Ni proton ni neutron se ne raspadaju u slobodnom stanju.

6. Da li su jezgre čestice ili valovi?

a) Jezgre su isključivo čestice jer im je masa dovoljno velika.

b) Jezgre su isključivo valovi jer se mogu kretati brzinom svjetlosti.

c) Jezgre su i čestice i valovi, ali njihove valne duljine nije moguće odrediti nikakvim

eksperimentom.

d) Jezgre su i čestice i valovi, već ovisno o tome kako interagiraju.

e) Jezgre se ne mogu nikad opisati ni kao čestice ni kao valovi.

7. Zaokruţite točnu tvrdnju vezanu uz gustoću atoma?

a) Gustoća atoma je reda veličine svakodnevih gustoća.

b) Gustoća atoma je otprilike 1 red veličine veća od reda veličine svakodnevnih gustoća.

c) Gustoća atoma je 1 red veličine manja od reda veličine svakodnevnih gustoća.

d) Gustoća atoma je 3 reda veličine manja od reda veličine svakodnevnih gustoća.

e) Gustoća atoma je 5 redova veličine manja od reda veličine svakodnevnih gustoća.

8. Ako su atomi tako rijetki, kako to da ne moţemo rukom kroz stol?

a) Zbog jakog elektrostatskog odbijanja elektrona iz atoma stola i atoma ruke.

b) Jer se stol sastoji od ogromnog broja atoma koji su gusto pakirani.

c) Zbog odbijanja svih elektrostatskih sila koje su veće od sile kojom moţemo djelovati

rukom.

d) Jer su sile meĎu atomima dovoljno jake da nas spriječe da proĎemo rukom kroz stol.

e) Nijedan od gornjih odgovora nije točan.

9. Da li je moguće da elektron vezan uz jezgru s puno naboja u nju uĎe?

a) Nije moguće. Elektron ima samo dozvoljena stanja van jezgre.

b) Nije moguće zbog jakih nuklearnih sila koje odbijaju elektron.

c) Nije moguće. Elektron se moţe nalaziti samo oko jezgre, kako god blizu ili daleko.

d) Moguće je. Elektron se moţe naći u jezgri iako je vjerojatnost za to mala.

e) Moguće je zbog jakih nuklearnih sila koje privlače elektron.

Page 42: implementacija konceptualnog testa iz nuklearne fizike i fizike čestica

37

10. Kako to da su jezgre stabilne ako se protoni elektrostatski odbijaju?

a) Elektrostatska Coulombova sila ne djeluje unutar jezgre.

b) Elektrostatska sila je uravnoteţena nuklearnom silom privlačenja.

c) Jezgra sadrţi jednak broj elektrona i protona pa se njihovo meĎudjelovanje neutralizira.

d) Neutroni u jezgri sprečavaju izravno meĎudjelovanje protona jednih na druge.

e) Moguće je ako se neutroni postave točno izmeĎu protona kako bi neutralizirali njihovu

silu odbijanja.

11. Da li radioaktivni izotop moţe emitirati samo jedan neutron?

a) Ne moţe, jer neutron nije ni α, ni β, ni γ zračenje.

b) Ne moţe, jer se time narušavaju neki zakoni očuvanja.

c) Ne moţe, jer to nikad nije energetski povoljen proces.

d) Moţe, iako se to rijetko dešava u odnosu na ostale tipove raspada.

e) Moţe, i to se dešava češće od ostalih tipova raspada.

12. Da li na radioaktivnost nekog elementa utječe malo povišenje njegove temperature?

a) Ne, ni u kakvoj mjeri.

b) Da, zato što sigurno na apsolutnoj nuli nema radioaktivnosti.

c) Da, izotopi više temperature puno jače zrače.

d) Da, izotopi više temperature puno slabije zrače.

e) Da, temperatura utječe na radioaktivnost, ali iznimno malo.

13. Zamislite sustav od 1000 radioaktivnih atoma. Koliko vremena poluţivota je prošlo kad se

raspalo 750 atoma?

a) Pola vremena poluţivota.

b) Jedno i pol vrijeme poluţivota.

c) Dva vremena poluţivota.

d) IzmeĎu dva i tri vremena pluţivota.

e) Četiri vremena poluţivota.

14. Radioaktivni element ima vrijeme poluraspada 10 minuta. U tom vremenskom intervalu se

neke od jezgara elementa ne raspadnu. Kolika je vjerojatnost da se u idućih 10 minuta

raspadnu i sve te jezgre?

a) Manja od 50%.

b) Veća od 50%.

c) Točno 50%.

d) Treba nacrtati graf eksponencijalne funkcije i očitati.

e) Ne moţe se odrediti.

Page 43: implementacija konceptualnog testa iz nuklearne fizike i fizike čestica

38

15. Kako se interpretira vrijeme poluţivota u sustavu jednog atoma?

a) Vrijeme nakon kojeg je vjerojatnost raspada 1/2.

b) Vrijeme u kojem je vjerojatnost raspada 1/2.

c) Ne moţe se definirati vrijeme poluţivota za samo jednu jezgru.

d) Besmisleno je govoriti o vremenu poluţivota za jedan atom, jer eksponencijalna funkcija

nikad ne doseţe nulu.

e) Vrijeme nakon kojeg se atom raspao, dijeljeno s 2.

16. Jedan radioaktivni atom ima vrijeme poluţivota 10 minuta. Ako se u tom vremenskom

intervalu atom ne raspadne, koja je od slijedećih tvrdnji istinita?

a) Nakon isteka vremena pluţivota, vjerojatnost raspada ostaje nepromijenjena.

b) Nakon isteka vremena poluţivota smanjuje se vjerojatnost raspada.

c) Nakon isteka vremena poluţivota povećava se vjerojatnost raspada.

d) Jedan atom će se sigurno raspasti nakon dva vremena poluţivota.

e) Jedan izdvojeni atom se nikada neće raspasti.

17. Pretpostavite da ste našli nekakav kamen koji je radioaktivan i moţe sadrţavati više

radioaktivnih elemenata. Da li se aktivnost tog kamena moţe povećavati s vremenom?

a) Ne, aktivnost kamena uvijek se smanjuje kvadratno s vremenom.

b) Ne, aktivnost kamena uvijek se smanjuje eksponencijalno s vremenom.

c) Ne, aktivnost kamena njegovo je temeljno svojstvo i ne mijenja se s vremenom.

d) Da, zato što aktivnost uvijek raste s vremenom.

e) Da, zato što ne znamo unaprijed od čega je sastavljen kamen.

18. Da li se jezgra moţe u jednom trenu (ne u lancu) raspasti na više od dvije čestice odjednom?

a) Moţe, to je prirodna stvar.

b) Moţe, ali samo u nuklearnim reaktorima.

c) Ne moţe, jer je energetski uvijek isplativije raspasti se na dva dijela.

d) Ne moţe zbog nekih zakona očuvanja.

e) Niti jedan od odgovora nije točan.

19. Zašto se prilikom raspada jezgre produkti raziĎu sa velikom kinetičkom energijom?

a) Jer ukupna energija prilikom raspada nije očuvana.

b) Jer masa pojedinih nukleona nije očuvana.

c) Ta energija je došla od energije vezanja.

d) Ta energija je došla od energije mirovanja.

e) Ne raziĎu se s velikom kinetičkom energijom ako sama jezgra prije raspada miruje.

Page 44: implementacija konceptualnog testa iz nuklearne fizike i fizike čestica

39

20. Foton valne duljine 500nm se pusti u gibanje od točke 1 do točke 2 kao na slici. Koji crteţ

najbolje odgovara putanji fotona?

21. Da li čestice bez naboja (npr. neutron) imaju antičestice?

a) Da, svaka čestica ima antičesticu.

b) Ne, jer nemaju naboj, pa nema što biti „anti“.

c) Da, jer iako nemaju naboj, sve su čestice sastavljene od manjih naboja.

d) Ne, jer samo elektroni i protoni imaju antičestice.

e) Ne, zato što antičestice ne postoje.

22. Ako bi postojao dio Svemira koji se sastoji od antičestica, kako bi tamo izgledao vodikov

atom i kako bi se ponašao?

a) Izgledao bi jednako kao i vodikov atom i imao bi ista svojstva.

b) Izgledao bi jednako kao i vodikov atom, ali ne bi imao ista svojstva.

c) Sastojao bi se od antiprotona i pozitrona, i imao bi ista svojstva.

d) Sastojao bi se od antiprotona i pozitrona, i ne bi imao ista svojstva.

e) Nemoguće je sastaviti takav „anti“-atom.

23. Da li zakon očuvanja mase vrijedi i u mikrosvijetu i u makrosvijetu?

a) Vrijedi samo u mikrosvijetu.

b) Vrijedi samo u makrosvijetu.

c) Vrijedi i u mikrosvijetu i u makrosvijetu.

d) Ne vrijedi ni u mikrosvijetu, ni u makrosvijetu.

e) Masa nije stvarna veličina, pa se ne moţe definirati takav zakon.

Page 45: implementacija konceptualnog testa iz nuklearne fizike i fizike čestica

40

24. Za što su zasluţne jaka i slaba sila?

a) Jaka sila drţi jezgru na okupu, a slaba nukleone.

b) Slaba sila drţi jezgru na okupu, a jaka nukleone.

c) Jaka sila drţi na okupu i jezgru i nukleone, a slaba je zasluţna za nešto drugo.

d) Slaba sila drţi na okupu i jezgru i nukleone, a jaka je zasluţna za nešto drugo.

e) Niti jedan od odgovora nije točan.

25. Koja od fundamentalnih sila je zasluţna za trenje?

a) Gravitacijska.

b) Elektromagnetska.

c) Jaka.

d) Slaba.

e) Niti jedna, jer je trenje nekozervativna sila.

26. Da li sila djeluje trenutno ili joj treba neko vrijeme?

a) Sila djeluje trenutno.

b) Signal putuje brzinom većom od brzine svjetlosti, ali sila ne djeluje trenutno.

c) Signal putuje brzinom jednakoj brzini svjetlosti.

d) Signal putuje brzinom jednakoj ili manjoj od brzine svjetlosti.

e) Niti jedan od odgovora nije točan.

27. Je li foton djeljiv?

a) Ne, apsolutno ga je nemoguće podijeliti.

b) Moţe ga se podijeliti samo u nekim slučajevima.

c) Svaka je čestica djeljiva – samo se mora razbiti dovoljno velikom energijom.

d) Sve čestice osim fotona su djeljive.

e) Niti jedan od odgovora nije točan.

28. Neka čestica sastoji se od kvarka i antikvarka. Što se sa njima desi kad se čestica raspadne?

a) Sastavnice anihiliraju i masa prelazi u energiju točno jednog fotona.

b) Sastavnice anihiliraju i masa prelazi u energiju točno dva fotona.

c) Sastavnice anihiliraju i masa prelazi u energiju jednog ili dva fotona.

d) Sastavnice anihiliraju i masa prelazi u energiju dva ili više fotona.

e) Sastavnice anihiliraju i masa prelazi u energiju bilo kojeg broja bilo kakvih čestica.


Top Related